Сохранен 536
https://2ch.hk/sci/res/320047.html
24 декабря Архивач восстановлен после серьёзной аварии. К сожалению, значительная часть сохранённых изображений и видео была потеряна. Подробности случившегося. Мы призываем всех неравнодушных помочь нам с восстановлением утраченного контента!

Математика, тред 37

 Аноним 20/12/15 Вск 06:07:05 #1 №320047 
14505808256150.png
Гуглопереводчик переводит "pointless topology" как "бессмысленная топология".

Предыдущий: >>317156 (OP)
Архивы Аноним 20/12/15 Вск 06:09:24 #2 №320049 
30. http://arhivach.org/thread/114111/
31. https://arhivach.org/thread/116099/
32. https://arhivach.org/thread/118093/
33v1. https://arhivach.org/thread/122613/
33v2. https://arhivach.org/thread/122615/
34. https://arhivach.org/thread/123717/
35. https://arhivach.org/thread/128822/
36. https://arhivach.org/thread/129039/
Аноним 20/12/15 Вск 08:05:11 #3 №320053 
а что за книга?
Аноним 20/12/15 Вск 10:09:56 #4 №320060 
>>320053
Голдблатт.
Аноним 20/12/15 Вск 13:42:57 #5 №320081 
14506081774740.png
>>320047 (OP)
Гуглопереводчик прав.
Аноним 20/12/15 Вск 13:53:26 #6 №320082 
>>320081
Что за ересь на пике?
Аноним 20/12/15 Вск 14:21:22 #7 №320086 
14506104824840.png
Ентри левел.

Подскажите, как пикрелейтед на множители разложить? С a^4+a^2+1 справляюсь, а с этим нет.

Есть годная литература по подобным элементарным преобразованиям?
Аноним 20/12/15 Вск 14:34:30 #8 №320089 
>>320086
Положи a2 = t и разложи как t4+t2+1.
Аноним 20/12/15 Вск 15:12:02 #9 №320091 
>>320082
Апофеоз ультрафинитизма.
Аноним 20/12/15 Вск 15:18:59 #10 №320092 
>>320086
a^4=t.
t^2 + t + 1 = 0.
t^2+t+1+t-t=(t+1)^2 - t=t^2(1+1/t)^2 -t=t(t(1+1\t)^2 - 1)=t( (t(1+1\t)+1)(t(1+1\t) -1) ).
Аноним 20/12/15 Вск 15:20:01 #11 №320093 
>>320092
Не спрашивай, к чему вторая строчка. Я не знаю.
Аноним 20/12/15 Вск 15:22:45 #12 №320094 
>>320093
и третья тоже
Аноним 20/12/15 Вск 15:30:57 #13 №320095 
>>320092
>(t+1)^2 - t
(t+1 - sqrt(t))(t+1 + sqrt(t))
Аноним 20/12/15 Вск 17:54:40 #14 №320112 
14506232807470.png
>>320086
Аноним 20/12/15 Вск 18:18:56 #15 №320116 
>>320086
>>320112
Алсо, я не знаю как без комплексных чисел разложить.
> Есть годная литература по подобным элементарным преобразованиям?
Просто изучай алгебру. Делать подобные преобразования с помощью школьных методов - это тупое олимпиадное говно, которое не имеет никакого отношения к математике.
Аноним 20/12/15 Вск 18:22:04 #16 №320117 
>>320112
Спасибо! Наконец-то понял. Я вообще ноль в подобных преобразованиях - знал, что суть разложения завязана на корнях, но реализовать не получалось.
Аноним 20/12/15 Вск 18:24:49 #17 №320118 
А теория вероятностей это картофан?
Аноним 20/12/15 Вск 18:37:44 #18 №320123 
>>320118
Нет (если строго к ней подходить).
Аноним 20/12/15 Вск 18:41:21 #19 №320126 
14506260813740.png
Как в вольфраме построить график системы с параметром? хуй знает как это правильно называется В общем пик.
Аноним 20/12/15 Вск 18:56:10 #20 №320131 
>>320126
С помощью ключевого слова "solve".
http://www.wolframalpha.com/input/?i=solve+x%3D%28t-1%29%5E2%2F2%2C+y%3D%28t%2B1%29%5E2%2F2
Аноним 20/12/15 Вск 18:57:57 #21 №320132 
>>320131
Воу, кстати просто через точку с запятой ; также строит.
Аноним 20/12/15 Вск 20:56:22 #22 №320171 
Существует ли удобная программа для Windows для записи математических формул с помощью клавиатуры?
Аноним 20/12/15 Вск 21:01:49 #23 №320174 
>>320171
Latex (Texmaker)
Аноним 20/12/15 Вск 21:04:34 #24 №320176 
Помогите насчет наклонных ассимптот.
У одногруппника предел который k - бесконечность, а b равен 0. Ну и он пишет, что ассимптота y = 2x. А у меня k = 1, а b бесконечность. По идее должны оба предела быть конечными? Но у него вроде препод проверила и норм всё. Моча какая-та.
Аноним 20/12/15 Вск 22:25:32 #25 №320186 
>>320176
реально моча, потому что если b бесконечность то асимптота твоя начинается на бесконечности, дурачёк чтоле?
Аноним 20/12/15 Вск 22:26:20 #26 №320187 
>>320176
и у него тоже моча. показывай пример
Аноним 20/12/15 Вск 22:27:45 #27 №320189 
>>320187
>>320186
Щас скину, сядь погоди. Еще у меня непонятка с вогнутостью
Аноним 20/12/15 Вск 22:29:23 #28 №320190 
>>320187
>>320186
А не, ну я прочувствовал, что одна асимпотота это когда оба предела к + беск., а другая, когда к минус. Это понятно, меня другая проблема больше волнует.. секунду
Аноним 20/12/15 Вск 22:33:10 #29 №320196 
14506399907140.png
14506399907301.png
Вот мое говно. Вроде график показывает выпуклость вниз, а производная наоборот. И у одногруппника также (второй пик)
Аноним 20/12/15 Вск 22:41:18 #30 №320202 
>>320196
Общее практическое правило:

Если существуют два конечных предела Формулы нахождения наклонной асимптоты, то прямая является наклонной асимптотой графика функции при . Если хотя бы один из перечисленных пределов бесконечен, то наклонная асимптота отсутствует.
http://www.mathprofi.ru/asimptoty_grafika_funkcii.html
Аноним 20/12/15 Вск 22:48:33 #31 №320205 
>>320202
Вроде понятно.
А про выпуклость что скажешь?
Аноним 20/12/15 Вск 23:10:30 #32 №320214 
>>320205
если всюду на интервале f''(x)>0 , то функция имеет вогнутость на этом интервале, если f''(x)<0, то функция имеет выпуклость.
Аноним 20/12/15 Вск 23:13:44 #33 №320216 
>>320214
Епты, я знаю определение вогнусти и выпуклости. Чому с гграфиком не сходится?
Аноним 20/12/15 Вск 23:22:09 #34 №320218 
>>320216
> Чому с гграфиком не сходится?
тому, что у тебя f(x) - это нихуя не функция
Аноним 20/12/15 Вск 23:27:13 #35 №320219 
>>320218
А че там?
Вообще я сам ебанулся, когда вольфрам построил это дерьмо. То есть всё более-менне правильно?
А как от руки это строить?
Аноним 20/12/15 Вск 23:29:19 #36 №320220 
>>320216
матан же простой как три рубля, че там с этими правилами исследования функции непонятного? ты кинул какой то кусок, там даже толком второй производной не видно, не то что функции. и график хуй пойми чего. чего с чем не сходится? вообще охуеть)
Аноним 20/12/15 Вск 23:31:43 #37 №320221 
>>320219
ебать, да фольфрамом это дерьмо строить сложнее чем от руки. с помощью этих правил анализа функции не думая вычисляешь особые точки, и соединяешь их, учитывая выпклость и вогнутость. а вольфрамом он это говно просто вычислил
Аноним 20/12/15 Вск 23:37:04 #38 №320224 
14506438240480.png
>>320221
>>320220
Оой, ну там в общем параметрическая функция. Я типа смотрел на x(t), потом на y(t), потом нашел производную y'(x) = y'(t)/x'(t), и там в общем тоже говно непонятное, типа функция всегда возрасатет. Потом вторая производная (на пике есть), там тоже говно, что всегда вогнутая. Как это вообще строить-то, если точек нет?
Аноним 20/12/15 Вск 23:55:25 #39 №320227 
>>320224
почему нет? они есть. просто заданы по другому. но есть. так и строить. просто это её так через жопу задали. и в том своде правил для анализа функций была помню специально оговорка о том как поступать когда функия задана параметрически
Аноним 21/12/15 Пнд 00:23:32 #40 №320231 
>>320081
просто кучу недомемасов типа 0.999...=1, -1/12, е в степени и пи = -1 напихали и смешно тип
Аноним 21/12/15 Пнд 00:25:38 #41 №320232 
>>320227
> просто это её так через жопу задали. и в том своде правил для анализа функций была помню специально оговорка о том как поступать когда функия задана параметрически
proigral s kartfanov
Аноним 21/12/15 Пнд 08:29:04 #42 №320297 
>>320232
в каком смысле?как быть когда ты например измеряешь силу тока, в зависимости от напряжения для какого то нелинейного элемента. и тут оказывается что напряжение от времени твое меняется по синусу, а ток все время растет, но очень медленно. в таком случае ты и напряжение и ток будешь задавать через время. а не друг через друга. тот самый случай задания функции параметрически. с одной стороны что х что у в каждый момент времени определены, а значит есть точки. с другой х от у выразить в данном случае геморно, поэтому выразили вот так. но список правил для анализа функции запилили и для этого случая. там были какие то простенькие оговорки, мол производные надо таким то образом брать. чтоб получить инфу о поведении функции на каждом из интервалов, между волшебными точками. ты сам эти правила прочитаешь или пошлешь меня нахуй за пруфами? хотя я и не пойду никуда - не нравится мое обьяснение - можешь не верить. один хер сессию с таким подходом не сдашь
Аноним 21/12/15 Пнд 10:50:21 #43 №320317 
>>320196
Чет вспомнил, как в школе этой хуйней занимался. Нахуя эти правила построения графиков нужны? Берешь и по точкам строишь в любом мат пакете.
Аноним 21/12/15 Пнд 11:00:21 #44 №320320 
>>320317
эти правила придумали задолго то того как можно было это построить по точкам. более того эти правила это как бэ суть подхода по исследованию какой угодно функции в каких угодно точках. и в процессе их запила вывели много чего полезного. например то же правило лопиталя, без которого делить бескончно малые и бесконечно большие величины трудно. а еще яснее все станет на 2 м курсе вузика
Аноним 21/12/15 Пнд 11:08:13 #45 №320323 
вообще строить именно эти графики с помощью программ, а не по этим правилам, это как считать 9*9 на калькуляторе и спрашивать - зачем нужна таблица умножения? калькулятор же быстрее считает. так и хочется ответить - да ни зачем, считай на калькуляторе. и это действительно будет лучший ответ, если спрашивающий - гуманитарий. гуманитарию это дерьмо нахер не нужно. и не в контексте "гуманитарий не человек", а действительно нахер не нужно. но ты же не гуманитарий? вот за тем же, зачем и таблица умножения эти правила нужны
Аноним 21/12/15 Пнд 12:51:51 #46 №320333 
Можно ли в вольфраме графы строить?
Аноним 21/12/15 Пнд 12:54:01 #47 №320334 
Почему Кадомцев-Петриашвили новый мем? Это только у нас так?
Аноним 21/12/15 Пнд 13:14:21 #48 №320337 
Матаны, к вам пара вопросов. Если вас не затруднит, конечно.

1. Обязательно ли в пространстве координатные оси всех измерений должны быть ортогональны?

2. Почему сначала выполняются операция умножения, а уже затем сложения? Это откуда-то следует? Или просто аксиома?
Аноним 21/12/15 Пнд 13:20:08 #49 №320338 
>>320337
не обязательно. вот цилиндрическая например https://ru.wikipedia.org/wiki/%D0%A6%D0%B8%D0%BB%D0%B8%D0%BD%D0%B4%D1%80%D0%B8%D1%87%D0%B5%D1%81%D0%BA%D0%B0%D1%8F_%D1%81%D0%B8%D1%81%D1%82%D0%B5%D0%BC%D0%B0_%D0%BA%D0%BE%D0%BE%D1%80%D0%B4%D0%B8%D0%BD%D0%B0%D1%82 . там вместо одной оси вообще угол. зачем они нужны, эти изъебские системы координат? некоторое говно, например электромагнитные поля и волны бывает проще считать в таких вот изъебских системах. там куча всяких коэффициентов уходит. про второй вопрос вообще нихера не понял. умножение это сокращенная запись сложения. это как спрашивать почему перед ночью бывает день - это откуда то следует?
Аноним 21/12/15 Пнд 13:27:10 #50 №320340 
>>320338
предвосхищая кукарекание, что ортогональность тут все таки выполняется, на так называемый общий случай - криволинейные системы координат https://ru.wikipedia.org/wiki/%D0%9A%D1%80%D0%B8%D0%B2%D0%BE%D0%BB%D0%B8%D0%BD%D0%B5%D0%B9%D0%BD%D0%B0%D1%8F_%D1%81%D0%B8%D1%81%D1%82%D0%B5%D0%BC%D0%B0_%D0%BA%D0%BE%D0%BE%D1%80%D0%B4%D0%B8%D0%BD%D0%B0%D1%82 там тензорным языком обьясняется всё.
Аноним 21/12/15 Пнд 14:09:21 #51 №320344 
>>320340
почему бы тензорным же языком не объяснить и в других ск ? Ты знаком с понятием "тензор", друг мой ?
Аноним 21/12/15 Пнд 14:14:31 #52 №320345 
>>320337
1) полечись от слабоумия, скушав мышьяка
2) если ты жив см п.1
Аноним 21/12/15 Пнд 14:38:32 #53 №320347 
>>320344
знаком. только не пойму к чему этот вопрос? сейчас будет "кудах тах тах, врёти"? простым языком я обьяснил. при желании вскукарекнуть что простым языком не описаны какие то нюансы я дал более полное формальное обьяснение. оно через тензоры. этот твой вопрос это попытка чего? сказать мне что я не в курсе про тензоры и поэтому все то обьяснение, что я привел полное говно? расстрою - дела обстоят именно так. по крайней мере так считает научное сообщество, и переходить на личности (знаком ли тензор лично мне) тут немного глупо. если говорить о лично моих знаниях, умении доносить мысль и так далее - я дипломированный инженер связи. по крайней мере те, перед кем я защищал свой диплом сочли, что я достоин звания инженера связи. в том числе и в вопросе математики. и да, именно из-за случаев, когда применение косоебаных систем отсчета оправдано я как бэ в курсе что такие системы есть и их широко применяют.
> почему бы тензорным же языком не объяснить и в других ск ?
а почему бы в задаче когда два поезда едут друг навстречу другу не считать их скорости друг относительно друга не по галилею а по лоренцу? потому что 1) это излишне загромаздит решение задачи переменными, которые будут в данном случае бесконечно малы и их можно не учитывать 2) тому кто спрашивает как решать задачу с двумя паровозами даже вредно начинать втирать про лоренца - только сильнее запутается. всем же в школе математику начинают преподавать далеко не с интегралов. тут так же
Аноним 21/12/15 Пнд 15:32:11 #54 №320353 
>>320347
но вот когда мы захотим понять, а с фига ли собственно, а не "посчитать на калькуляторе 9*9" таки придется. Не считаешь так ?
Почему сразу не сказать, мол, скушай теорию, хорошо пережевав и сглотнув, а не вдыхай сладостный аромат доказанного, не готовив его сам ?
Аноним 21/12/15 Пнд 16:31:37 #55 №320357 
>>320353
я не пойму вопроса - чувак спросил. судя по тому как спросил, сам он не в курсе. ответить ему - иди учись сука, тогда поймешь? возможно он и так учится, и дальше узнает это всё сам. а сейчас хочет одним глазком глянуть. вот поглядел. а может и не учится, а так чисто праздный интерес. и опять же, почему бы его не удовлетворить - узнал немного нового да и хер забил - тогда и подводить всю теоретическую базу бессмыслено, будет как бисер перед свиньями. но в любом случае по мне так лучше обьяснить человеку хоть как то, чем просто посылать его нахуй, говоря азаза школота иди нахуй. если же парень захочет очень глубоко погрузится во все эти коши с гауссами, он всегда сможет ознакомится с конкретными изданиями. не вижу ничего плохого в том, чтобы обьяснять что кирпич красный потому, что все остальные длины волн поглащаются его атомами. а не посылать чувака учить квантмех. или даже общую физику. это просто излишне.
Аноним 21/12/15 Пнд 17:18:15 #56 №320379 
Анон, напомни, как строится минимальный многочлен элемента конечного поля? Надо составить произведения вида (x - a)(a- a^n)... и т.д. Вот эти степени элемента a как вычисляются? Реквестирую теорию.
Аноним 21/12/15 Пнд 18:11:45 #57 №320391 
>>320338
Спасибо, анон.

А на счет второго пункта, мне интересно почему, например, в выражении 2+2*2 ответ 6, а не 8. На уровне банальной повседневной логики я это понимаю, но мне интересно есть ли у этого правила порядка выполнения действий строгая математическая формулировка.
Аноним 21/12/15 Пнд 19:25:59 #58 №320401 
>>320323
> это как считать 9*9 на калькуляторе и спрашивать - зачем нужна таблица умножения?
А она и не нужна. Ну, может кондуктору нужна, или продавцу. А математику зачем уметь умножать числа больше 5?
Аноним 21/12/15 Пнд 20:22:59 #59 №320424 
>>320401
Ну так, сдачу посчитать, например. В современных социально-экономических реалиях, между прочим, вещь очень полезная.
Аноним 21/12/15 Пнд 20:29:03 #60 №320425 
>>320424
На чеке пишут все товары, тебя никак не наебут.
Аноним 21/12/15 Пнд 20:35:21 #61 №320427 
>>320425
Сдачу не на всех чеках пишут. А бабки на рынках чеков вообще не выдают.
Аноним 21/12/15 Пнд 21:05:44 #62 №320428 
Сап, аноны. Учусь на мехмате, 4 курс. Решил освежить в памяти численные методы. Посоветуйте хороших книг по теме, пжлст. С меня как обычно
Аноним 21/12/15 Пнд 22:39:20 #63 №320442 
14507267608570.png
>>320047 (OP)
Можно простым языком как так или простой пример где так же делают?
Аноним 21/12/15 Пнд 23:14:36 #64 №320450 
14507288768950.png
Посоны, сижу тут делаю ангем и внезапно встреваю в интересное положение. По алгоритму надо перенести остаточный член из последней строки пикрелейтеда вправо и поделить на него. Но у меня он равен 0! На 0 что ли делать? В методичке такого нет!
Аноним 21/12/15 Пнд 23:41:10 #65 №320451 
14507304704280.jpg
>>320047 (OP)
Мочизуки смотрит на вас как на театр Ходжа
Аноним 22/12/15 Втр 00:10:54 #66 №320455 
14507322544870.png
Что выгоднее бросать 5 кубов, которые успешны на 5 и 6, ИЛИ 2 куба, которые успешны на 4, 5, 6?
Еще было бы неплохо, если поясните как это считать.
Спасибо.
Аноним 22/12/15 Втр 00:45:32 #67 №320460 
>>320401
>А математику зачем уметь умножать числа больше 5?
спроси у математика знакомого, он обьяснит доходчивее
Аноним 22/12/15 Втр 01:00:41 #68 №320463 
>>320425
> не наебут
Если ты умеешь считать.
Аноним 22/12/15 Втр 01:51:06 #69 №320470 
>>320428
бамп
Аноним 22/12/15 Втр 05:57:37 #70 №320483 
>>320460
Я математик. Последний раз два числа умножал в прошлом году, когда помогал анону с домашкой.
Аноним 22/12/15 Втр 09:29:56 #71 №320495 
>>320337
> Почему сначала выполняются операция умножения, а уже затем сложения
просто потому что так договорились
Аноним 22/12/15 Втр 09:57:40 #72 №320498 
>>320483
Чем занимаешься?
Аноним 22/12/15 Втр 10:01:10 #73 №320499 
>>320455
Посчитай неуспешность.
Аноним 22/12/15 Втр 11:20:45 #74 №320504 
>>320391
а, я сперва не понял. это здравый смысл чтоле. ну типа допустим что это не так, и можно делать любые действия. 2+33. 33 это сокрашенная запись 3+3+3. если ты сперва сложишь 2+3 то получишь 5*3. а это будет уже 3+3+3+3+3. но это ведь херня. ну тоесть про очередность выполнения действий это не то что аксиома, просто скажем так правило, позволяющее не задумываться каждый раз о том что умножение это сложение, не расписывать всё гавно, а быстренько посчитать. ну типа как формулы сокрашенного умножения - если их знаешь то быстрее решишь какой нить пример. не зная их ты их всегда можешь вывести, а зная даж задумываться об этом не надо, можно думать о задаче в целом например
Аноним 22/12/15 Втр 12:56:41 #75 №320523 
>>320504
>2+33, почему то отклеилась
Аноним 22/12/15 Втр 12:57:45 #76 №320524 
2+3 умножить на 3 ебаная макаба блять. почему то звездочку сжирает
Аноним 22/12/15 Втр 14:35:55 #77 №320544 
>>320499
Ничего не понимаю в матане, поясните как подсчитать или подсчитайте за меня.
Спасибо.
Аноним 22/12/15 Втр 15:38:34 #78 №320554 
>>320455
в первом случае у тебя успех 2 из 6 комбинаций для одного кубика. не оговорено кстате - достаточно чтоб на хотя бы одном выпал или на всех? во втором случае 3 из 6 для одного кубика. 1)оговори на скольких кубиках должна выпасть удача 2) посчитай чесло этих комбинаций.
>>320544
сижу на форуме где задают вопросы.
посылаю нахуй тех кто их задает.
ощущаю свою никчемность.
Аноним 22/12/15 Втр 16:23:49 #79 №320559 
>>320498
Сидит в вербитятнике, дрочит на аниму, постит Мочидзуку, известное дело. Это же математик уровня sci.
Аноним 22/12/15 Втр 19:50:27 #80 №320595 
>>320504
> 2+33. 33 это сокрашенная запись 3+3+3. если ты сперва сложишь 2+3 то получишь 5*3. а это будет уже 3+3+3+3+3.
Ты правда считаешь, что это объяснение? А почему тогда 2 + sqrt(3) x sqrt(2) не равно (2 + sqrt(3)) x sqrt(2)?

>>320391
У каждой операции есть приоритет. При вычислении значения выражения операции выполняются в порядке убывания приоритета. Умножение имеет более высокий приоритет чем сложение.
Аноним 22/12/15 Втр 20:06:17 #81 №320598 
>>320337
> 1. Обязательно ли в пространстве координатные оси всех измерений должны быть ортогональны?
Что ты имеешь в виду под координатными осями? Если, например, взять линейное пространство и координатными осями называть вектора базиса, то они могут быть неортогональны.
Аноним 22/12/15 Втр 20:17:37 #82 №320599 
>>320595
>Ты правда считаешь, что это объяснение?
да, а ты его наверное не понял нихуя)
>А почему тогда 2 + sqrt(3) x sqrt(2) не равно (2 + sqrt(3)) x sqrt(2)?
именно по моему обьяснению
Аноним 22/12/15 Втр 20:23:48 #83 №320600 
>>320599
Ну распиши мне тогда sqrt(3) x sqrt(2) через сложение.
Аноним 22/12/15 Втр 20:30:59 #84 №320601 
>>320600
Да легко.
Аноним 22/12/15 Втр 20:31:51 #85 №320602 
>>320600
в каком классе, дружок? распиши мне до последней цифры диагональ квадрата со стороной 1 см. а в след году ты будешь проходить логарифмы с такой херней в основании. а если даже её осилишь, то поступишь в вузик и там, если опять же будешь хорошо учится, тебе расскажут про комплексные числа, дифференциальные уравнения и много другого в этом чудесном мире математики. твоя пошлая иррациональность слишком школьная. и вот когда тебя будут ебать всей этой херней ты поймешь наконец, что нет блять никаких "приоритетов операций" ослина. есть четкое определение операций и чисел. какая то аксиоматика. и на ней строятся теоремы и прочая хуйня. теоремы и прочая хуйня выводятся из аксиом. в аксиомах нету про приоритет умножения. есть что умножение==сокрашенная запись сложения. а теперь иди порешай квадратные уравнения и успокойся
Аноним 22/12/15 Втр 20:40:42 #86 №320606 
>>320602
> распиши мне до последней цифры диагональ квадрата со стороной 1 см
Это невозможно.

> умножение==сокрашенная запись сложения
Это означает, что sqrt(3) x sqrt(2) можно записать через сложение. Запиши.
Аноним 22/12/15 Втр 20:45:38 #87 №320608 
>>320606
почему невозможно? можно же написать 2^1/2? эта запись расписывает до любого приближения. вот и запись sqrt(3) x sqrt(2) расписывает для любого. конечно можно это представить и в виде ряда какого нить, но лень даже думать об этом
Аноним 22/12/15 Втр 20:47:02 #88 №320609 
Здравия желаю, матаны. Как найти энную производную 1/(1+x^2)? Интересует сам алгоритм. Комплексные числа использовать запрещено (работаем в R).
Аноним 22/12/15 Втр 20:47:57 #89 №320610 
>>320606
>sqrt(3) x sqrt(2)
даже еще смешнее - это ваше деление, которое тут "неявно" представлено, тоже ведь через сложение определяется, так что можно записать и через сложение тупо, но реально лень искать определение корня через ряды
Аноним 22/12/15 Втр 20:48:52 #90 №320611 
>>320609
а че в ней такого, что не канают таблицы производных?
Аноним 22/12/15 Втр 20:58:02 #91 №320614 
>>320608
> почему невозможно?
Потому что sqrt(2) не имеет "последней" цифры.

> вот и запись sqrt(3) x sqrt(2) расписывает для любого
Меня не интересует приближение. Меня интересует точный ответ.
Аноним 22/12/15 Втр 21:07:18 #92 №320617 
14508076381320.jpg
>>320602
Схоронил пасту
Аноним 22/12/15 Втр 21:08:38 #93 №320618 
>>320608
Я даже не знаю, предложить тебе дальше позориться или дать ссылку на определение умножения.
Аноним 22/12/15 Втр 21:12:10 #94 №320621 
>>320614
>Меня не интересует приближение. Меня интересует точный ответ.
совсем не знаком с мат анализом, правда?
>бесконечно точное приблежение==точный ответ, ибо 0,(9)==1 и так далее и на то похожее. эх, мне бы твои годы, всё еще в перде
Аноним 22/12/15 Втр 21:13:00 #95 №320622 
Народ, есть условие в духе "даны четыре вектора a1 a2 a3 b с коорд. x1y1z1 и т.д. заданные в базисе e1e2e3. Покажите что a1 a2 a3 образуют базис. найдите коорд b в этом базисе" объясните как делать. Я вроде не прям уж чтоб совсем долбоёб, прост мож чего не понимаю.
Аноним 22/12/15 Втр 21:13:40 #96 №320625 
>>320618
я тебе сам её дам
Умноже́ние — одно из четырёх основных арифметических действий, бинарная математическая операция сложения указанного количества одинаковых слагаемых.
>сложения указанного количества одинаковых слагаемых.
>сложения
https://ru.wikipedia.org/wiki/%D0%A3%D0%BC%D0%BD%D0%BE%D0%B6%D0%B5%D0%BD%D0%B8%D0%B5
Аноним 22/12/15 Втр 21:15:05 #97 №320628 
>>320622
НУ ПОЖАЛЙУСТА ПОМОГИТЕ! ПИЗДЕЦ ДРОЧЬ ВЕДЬ КАКОЙ-ТО!
Аноним 22/12/15 Втр 21:15:15 #98 №320629 
>>320622
не понимаешь - гугли определение базиса и переход к новому базису. сразу всё поймешь
Аноним 22/12/15 Втр 21:18:03 #99 №320631 
14508082830580.jpg
>>320628
у тебя все получится
Аноним 22/12/15 Втр 21:18:56 #100 №320632 
>>320631
охуеть... пасибо
Аноним 22/12/15 Втр 21:25:30 #101 №320634 
>>320625
А ничего, что это определение только для натуральных чисел?

>>320621
>бесконечно точное приблежение==точный ответ, ибо 0,(9)==1 и так далее и на то похожее. эх, мне бы твои годы, всё еще в перде
Я не вижу никаких нормальных объяснений. Только махания руками про какие-то бесконечно точные приближения вместо нормального определения умножения.

>>320622
> объясните как делать
Тупо по определению. Список векторов образует базис, если они вектора линейно независимы и длина списка равна размерности пространства. То есть тебе надо показать, что a1, a2, a3 линейно независимы.
Аноним 22/12/15 Втр 21:31:48 #102 №320636 
>>320629
блядь, анон, ты как знал какую книжку я найду. всё прям как по оглавлению.
Аноним 22/12/15 Втр 21:34:12 #103 №320637 
>>320634
>Я не вижу никаких нормальных объяснений.
>Умноже́ние — одно из четырёх основных арифметических действий, бинарная математическая операция сложения указанного количества одинаковых слагаемых.

>А ничего, что это определение только для натуральных чисел?
вся беседа велась в их области. ты вводил иррациональные числа, для того чтобы вставить свой пример? ты и вводи определение умножения для них, да так чтобы оно сочеталось с умножением для натуральных чисел.или прооверяй\убеждайся что мое определение на них распространяется, мне похую
Аноним 22/12/15 Втр 21:39:53 #104 №320638 
>>320637
> >А ничего, что это определение только для натуральных чисел?
> вся беседа велась в их области.

> 33 это сокрашенная запись 3+3+3
> А почему тогда 2 + sqrt(3) x sqrt(2) не равно (2 + sqrt(3)) x sqrt(2)?
> именно по моему обьяснению
> умножение==сокрашенная запись сложения

> ты и вводи определение умножения для них, да так чтобы оно сочеталось с умножением для натуральных чисел
https://proofwiki.org/wiki/Definition:Multiplication/Real_Numbers

Пустил тугую струю урины в лицо картофану.
Аноним 22/12/15 Втр 21:42:56 #105 №320639 
>>320638
дурачек. а правая часть равенства как определена?
Аноним 22/12/15 Втр 21:45:14 #106 №320640 
>>320639
Очевидно, через рациональные числа. Но какая разница, если ты не можешь объяснить, почему a + b x c не равно (a + b) x c для вещественных a, b и c?
Аноним 22/12/15 Втр 21:47:20 #107 №320641 
>>320640
потому что первое действие всегда то, что в скобках
мимошкольник
Аноним 22/12/15 Втр 21:47:27 #108 №320642 
>>320640
конечно же по "приоритету операций". просто пиздец. лучший файлообменник нахуй
Аноним 22/12/15 Втр 21:56:42 #109 №320643 
>>320642
Ты серьезно считаешь, что "приоритет операций" не является общепринятым термином?
Аноним 22/12/15 Втр 22:05:51 #110 №320644 
>>320643
я серьезно считаю что он является следствием определения умножения через сложение
Аноним 22/12/15 Втр 22:06:50 #111 №320645 
>>320643
че ты будешь делать раньше - брать корень, синус, тангенс, экспоненту или логарифм?
Аноним 22/12/15 Втр 22:09:02 #112 №320646 
>>320611
Я не могу найти таблицу. А выводить, как ты понимаешь, не умею.
Аноним 22/12/15 Втр 22:16:05 #113 №320648 
>>320644
> я серьезно считаю что он является следствием определения умножения через сложение
Если считаешь так, то сформулируй определение умножения вещественных чисел и докажи.

>>320645
Зависит от выражения. Зачастую выражения написаны так, что компьютер не смог бы их проинтерпретировать однозначно. Например, log log n. Но человек может догадаться, что на самом деле имеется в виду.
Аноним 22/12/15 Втр 22:32:47 #114 №320650 
>>320648
>Зависит от выражения.
потому что определения этих операций не связаны друг с другом. были бы связаны, то был бы приоритет.
а сложение связано с умножением. и поэтому там есть приоритет.
>Если считаешь так, то сформулируй определение умножения вещественных чисел и докажи.
нет, это ты себе докажи. я себе это уже когда то доказал, а сейчас уже забыл. конечно за пол часа я нагуглю всю эту хуйню по новой, но я и так уже на примере синуса да логарифма показал отсутствие приоритета у не связанных функций. а дальше уже ты, если хочешь то докажи себе это, разберись так сказать в вопросе. или если не хочешь не разбирайся, останемся каждый при своем. не хочешь не верь мне. я ж уже говорил - мне похую
Аноним 22/12/15 Втр 22:34:38 #115 №320652 
>>320609
Бамп вопросу, аноны.
Аноним 22/12/15 Втр 22:37:46 #116 №320653 
>>320652
Для того, чтобы найти производную от частного двух функций необходимо, из произведения производной делимого, умноженной на функцию делителя, вычесть произведение производной делителя, умноженной на функцию делимого, и все это разделить на функцию делителя возведенную в квадрат. (u/v)' = (u'v-v'u)/v^2;
https://sites.google.com/a/nstuedu.com/perv/proizvodnye
Аноним 22/12/15 Втр 22:45:57 #117 №320655 
Если ты не используешь приоритет операций, то как ты определяешь, что означает "sin x^2" и "2n!"? Как вообще ты можешь вычислять значения выражений?
Аноним 22/12/15 Втр 22:52:26 #118 №320658 
>>320655
в обоих примерах двоякий контекст, формально без скобок такую запись можно прочитать и просчитать и так и сяк. а вот с 2+3 умножить на 3 двоякого толкования нет. строго сперва умножь, потом уже складывай. с синусом если ты не оговоришь скобками не ясно что делать сперва синус брать или в квадрат возводить. так же и со второй формой записи. точнее конечно для человека может и ясно, но формально это говно. типа скобок надо добавить. а в случае умножения не надо, в том чесле и потому, что это просто сокрашенная запись суммы в скобках
Аноним 22/12/15 Втр 23:00:07 #119 №320659 
>>320655
твоя проблема в том, что ты рассматриваешь sin и ln как функцию, а + и умножить нет. хотя что одна херь переводит изначальное поле значений в какое то другое(область значений и опрежделений синуса), так и + с умножить делают по сути те же операции -переводят одно поле чисел в другое. такая же функция. и тут ты видишь зависимость между функцией сложить и функцией умножить. а между другими аналогичными нет. значит одна == другая с другим коэффициентом. хотя че то меня понесло не туда. я из аксиомы(умножение==сокращенная запись сложения) тебе эту же аксиому доказываю. это высший идиотизм. за сим откланяюсь
Аноним 22/12/15 Втр 23:02:26 #120 №320661 
>>320653
И что? Там не выходит рекуррентность. Там выходит потом произведение двух функций. Там производная энного порядка не выражается.
Аноним 22/12/15 Втр 23:06:04 #121 №320664 
>>320661
прости - рекуррентность незавезли,самому придется придумать
Аноним 22/12/15 Втр 23:23:11 #122 №320667 
>>320658
Всегда если ты пишешь fx^k имеется в виду f(x^k), а не f в степени k или композиция f k раз. Потому что ^ имеет больший приоритет чем f.

Предположим, ты определил бинарную операцию #: R x R -> R и хочешь, чтобы выражение a # b x c означало a # (b x c). Более того, в общем случае у тебя есть E1 # E2, где E1 и E2 - произвольные выражения, и ты хочешь, чтоб порядок выполнения операций как-то зависел от E1 и E2. Как ты опишешь этот порядок не определяя приоритет операции #? Ты, конечно, можешь вместо этого всегда ставить скобки или вообще не использовать инфиксную форму и писать #(E1, E2), но это довольно хуево будет читаться.
Аноним 23/12/15 Срд 00:32:22 #123 №320675 
>>320611
>>320653
Его наверное интересует замкнутое выражение для любого n. Это задачка из самого начала тривиума Арнольда, так?
Аноним 23/12/15 Срд 02:44:55 #124 №320695 
Театр ходжа в хату, пацаны! Не заходил сюда год, сейчас пролистал тред, смотрю что-то стало всё не так: вместо натуральных чисел определяют умножение, мочиздука упоминается один раз, а навернуть теорию категорий пока что вообще не предлагали.

Но у меня к тебе есть просьба. Анон, накидай годных книг по вариационному исчислению, не страшны ни английский язык (ни даже французский, лол), ни высокая сложность книги. Желательно чего-нибудь поновее.
Аноним 23/12/15 Срд 04:52:32 #125 №320719 
>>320600
Допустим, натуральные числа со сложением у нас есть.
Умножение определяется как результат нескольких операций сложения.
Целые числа определяются как класс эквивалентности упорядоченных пар (a,b) натуральных чисел. Идея в том, что всякое целое число z может быть представлено в виде разности a-b, поэтому его можно отождествить с этой разностью; пары (3;5) и (7;9) эквивалентны. Таким образом, все действия в кольце целых чисел сводятся к действиям над натуральными числами.
Поле рациональных чисел определяется как поле частных целых чисел. Поэтому действия над рациональными числами сводятся к действиям над целыми и потому над натуральными числами.
Поле вещественных чисел определяется как пополнение поля рациональных чисел. Все действия над вещественными числами сводятся к действиям над последовательностями рациональных чисел и потому к действиям над натуральными числами. Хотя из-за непрерывности сложение даже двух вещественных чисел может потребовать бесконечного количества действий над натуральными числами.
Извлечение корня из вещественного числа может быть определено с помощью последовательностей.
Любую числовую операцию можно свести просто к некоторому количеству сложений натуральных чисел, возможно бесконечному.

>>320644
Нет, приоритет операций является просто договорённостью. Свидетельством этого служит лёгкость, с которой можно ввести альтернативный приоритет. Пример альтернативного - приоритет, в котором действия выполняются в том порядке, в котором идут знаки. Например, в альтернативном приоритете 2+2×2 = 8. Приоритет является просто соглашением по расстановке скобок, о чём писал ещё Клини.
Аноним 23/12/15 Срд 08:40:41 #126 №320725 
>>320667
возведение в степень есть сокрашенная запись умножения. и следовательно сложения. а синус с логарифмом нет. + >>320719
тебе не поленился и написал пасту для арифметики с вещественными. но еще раз - ты можешь не верить нам, а верить в свои приоритеты - нравится быть дураком - будь им
Аноним 23/12/15 Срд 11:27:30 #127 №320744 
Нахуя математика нужна?
Аноним 23/12/15 Срд 13:40:14 #128 №320761 
>>320744
Хуй на.
Каков вопрос - таков ответ.
Аноним 23/12/15 Срд 15:24:02 #129 №320790 
>>320744
Чтоб ебать твою мамку.
Аноним 23/12/15 Срд 15:38:03 #130 №320798 
>>320047 (OP)
ньюфаг в треде. На сегодняшний день уже придумали логическую концепцию, которая способна обойти парадокс Рассела?
Аноним 23/12/15 Срд 15:40:28 #131 №320799 
Посоны, а современные компьютерные вычисления в алгебре оформлены в отдельную дисциплину уже? Поделитесь книжечками? В частности, интересуют вычисления для нужд теории представлений.
Аноним 23/12/15 Срд 15:53:34 #132 №320802 
>>320798
Да. Парадокс Рассела возможен только в наивной теории множеств. В аксиоматической невозможен.
Аноним 23/12/15 Срд 16:14:19 #133 №320809 
>>320802
Угу, как же.
Аноним 23/12/15 Срд 19:29:01 #134 №320849 
>>320799
попробуй это глянуть
http://gen.lib.rus.ec/search.php?req=Representations+of+Groups+A+Computational+Approach&lg_topic=libgen&open=0&view=simple&phrase=1&column=def
Аноним 23/12/15 Срд 20:35:20 #135 №320865 
>>320849
Спасибо, выглядит интересно. Но, хотя я и не смотрел всё, там по-моему про сами алгоритмы ничего не сказано, а приведены инструкции для GAPа.
Аноним 23/12/15 Срд 22:20:33 #136 №320889 
14508984339190.png
>>320047 (OP)
Наверное нелепо просить в sci, но попрошу помощи. Нужно успеть сделать 3 задания до пятницы для зачета. Я бы с удовольствием сам посидел за этими заданиями, но совершенно нет времени, сегодня нужно готовиться к ангему, завтра вернусь домой только в 10. Буду очень благодарен хотя бы за основной путь решения.
Аноним 23/12/15 Срд 22:24:12 #137 №320890 
Посоны, в курсаче нужно было нарисовать фукнцию с пределом в точке (всё дано, кроме функции, лол). Ну я нарисовал. А потом преподша доебалась, типа у тебя не видна зависимость дельта от эпсилон. Хотя у меня на одном таки была отмечена дельта-окрестность и пунктирчиком прихуярено на график, а потом оттуда на oy. А другой график у меня с пределом в минус бесконечности, и я не понял нихуя как это нарисовать. По определению вроде есть некое E, которое всегда больше f(a). Ну я его не отметил, и короче бабка доебалась. Но я толком не понял из-за чего именно - она сказала что-то вроде "я могу любой эпсилон взять" или типа того не расслышал нихуя, в общем чего-то на графике не видно. Один график (который без бесконечности) я правильно сделал или нет? Одногруппники либо сдали и не могут нихуя объяснить, либо же с таким же обосрамсом.
Аноним 23/12/15 Срд 22:25:50 #138 №320891 
14508987507250.png
Вот фотка
>>320890
Аноним 23/12/15 Срд 23:05:26 #139 №320894 
>>320890
>>320891
Блядь, ты тут один с этим курсачом. Иначе, что это за хуйня с массовым наплывом курсовых по "исследованию функции" лол?
Аноним 23/12/15 Срд 23:08:20 #140 №320895 
>>320889
мехмат?
Аноним 23/12/15 Срд 23:08:29 #141 №320896 
>>320894
Да я сам уже охуел.
И вообще я не виноват, что у вас тут довольно тихо и я один воду мучу. Мне вообще желательно сдать эту хуйню завтра, тогда перестану вас доебывать.
Аноним 23/12/15 Срд 23:10:39 #142 №320897 
14509014390350.jpg
>>320895
Это у тебя такие шутки что ли?
Аноним 23/12/15 Срд 23:11:38 #143 №320898 
>>320897
да не, я серьёзно, по сравнению с этим >>320891 картофаном, твои задачки весьма сложные и интересные
Аноним 23/12/15 Срд 23:13:44 #144 №320899 
>>320898
http://www.youtube.com/watch?v=1oUlDTpeeiQ
Аноним 23/12/15 Срд 23:16:32 #145 №320900 
>>320898
Да, мехмат.
Аноним 23/12/15 Срд 23:18:49 #146 №320901 
>>320900
ну слава перуну, я думал, что я совсем тупой
первые две хуй знает как решить, третья похожа на теорему ролля (ну или лагранжа, все они там одинаковы по сути)
Аноним 23/12/15 Срд 23:20:06 #147 №320902 
>>320901
> третья похожа на теорему ролля (ну или лагранжа, все они там одинаковы по сути)
Как раз это самое изи, а вот с первыми двумя хотелось бы, чтобы анон помог.
Аноним 23/12/15 Срд 23:22:11 #148 №320903 
>>320902
вторая тоже изи, потому что если брать m = (p+q)^2 / 2, n = (p-q)^2 / 2, то из того выражения получаем любое рациональное число. Дальше сам.
Аноним 23/12/15 Срд 23:24:15 #149 №320904 
>>320903
бля, проебался, на 2 делить не надо, иначе не натуральные числа будут, конечно же
Аноним 23/12/15 Срд 23:24:55 #150 №320905 
>>320903
>>320904
Точно. Спасибо за идею.
Аноним 23/12/15 Срд 23:30:10 #151 №320906 
>>320905
первая задача тащемта похожа на подставу, потому что у функции sin x есть неподвижная точка, которая находится как раз на (0,pi)
Аноним 23/12/15 Срд 23:32:35 #152 №320907 
>>320906
В смысле неподвижная?
Аноним 23/12/15 Срд 23:33:44 #153 №320908 
>>320907
в прямом, больше не буду помогать
Аноним 23/12/15 Срд 23:35:56 #154 №320909 
>>320908
Хорошо. Большое спасибо тебе.
Аноним 24/12/15 Чтв 00:04:54 #155 №320914 
Если тыкнуть рандомное число на числовой прямой, оно 99,(9)% будет трансцендентным? Или так выбрать не получится из-за несчётности множества вещественных чисел?
Аноним 24/12/15 Чтв 00:07:28 #156 №320916 
>>320914
смотря что ты вкладываешь в понятие "тыкнуть рандомное число"
То есть нужна какая-то вероятностная мера на R. Часто она абсолютно непрерывна относительно меры Лебега, и в этом случае вероятность попасть в транцендентное число равна нулю.
Легко придумать такую вероятностную меру, для которой будет не ноль. Такие дела.
Аноним 24/12/15 Чтв 00:08:21 #157 №320917 
>>320916
равна 1, а не нулю объебался
Аноним 24/12/15 Чтв 12:02:54 #158 №320964 
>>320695
>а навернуть теорию категорий пока что вообще не предлагали.
А оп-пик тогда что?
Аноним 24/12/15 Чтв 15:16:22 #159 №321008 
Почему фундаментальная группа тора Z_2, а бублика не Z_4 ?
Как можно представить соотношение между независимыми петлями? Какая фундаментальная группа у сферы с n ручками?
Аноним 24/12/15 Чтв 15:29:25 #160 №321010 
>>320964
Объясните не математику что такое теория категорий.
Аноним 24/12/15 Чтв 15:45:28 #161 №321013 
Посоны, какое определение у предела при х стремящемся к беск., когда предел тоже равен бесконечности?
Аноним 24/12/15 Чтв 16:12:29 #162 №321018 
>>321013
ох уж эти картофанские проблемы
sageАноним 24/12/15 Чтв 16:29:43 #163 №321023 
>>321013
В любом учебнике по матанализу посмотри.
Аноним 24/12/15 Чтв 16:33:20 #164 №321024 
>>321010
Обычно когда ты пишешь отображение чего-то куда-то ты рисуешь буковки и стрелочки от одной буковки к другой. Теория категорий это наука о таких диаграммах из буковок и стрелочек.
Аноним 24/12/15 Чтв 16:34:42 #165 №321025 
>>321024
А что там научного, поясни, а? Там этих стрелочек может быть бесконечное число например?
Аноним 24/12/15 Чтв 18:14:32 #166 №321094 
>>321024
Ну не настолько для не математика. Чем она от теории групп отличается, например?
Аноним 24/12/15 Чтв 18:16:56 #167 №321095 
>>321094
Чем физика отличается от биологии, например?
Аноним 24/12/15 Чтв 18:17:17 #168 №321097 
>>321094
Чем футбол отличается от шахмат, например?
Аноним 24/12/15 Чтв 18:33:35 #169 №321101 
>>321095
>>321097
Не надо ерничать. Википедия говорит про группы как пример категорий. Просто объясните мне по-человечески, математики.
Аноним 24/12/15 Чтв 18:36:04 #170 №321103 
>>321101
>по-человечески
>математики
ЗАДАЧА ПО ТЕОРИИ ВЕРОЯТНОСТЕЙ Аноним 24/12/15 Чтв 19:06:33 #171 №321121 
Продублирую здесь, а то в /b/ смоет.

Аноны, как вы все знаете, скоро новый год, поэтому во всех офисах готовятся его отмечать. Некоторые нанимают тамаду, некоторые просто сидят и бухают, а некоторые, такие как наш офис, ищут в интернете всякие конкурсы и быдлоразвлекухи самостоятельно. Так вот один из таких конкурсов состоит в следующем: каждый член коллектива покупает небольшой подарок, заворачивает его в мешок (или кладет в коробочку) и скидывает в общую кучу. Потом, на празднике жизни, каждый по очереди тащит из мешка подарок (неизвестно чей). Сюрприз типа.
Конкурс, конечно, интересный, но вот незадача: а вдруг кому-нибудь попадется подарок, который он сам скинул? Неинтересно получается. Я задался вопросом: какова вероятность того, что каждый получит несвой подарок и все останутся счастливы? Хорошая задача по терверу, ящитаю.
Задача: вывести формулу (рекуррентную или аналитическую - не важно), которая бы позволила рассчитать вероятность удачного исхода, т.е. исхода, при котором никто не получит свой же подарок. Все подарки считать разными.
Помогите решить, аноны. Третий день голову ломаю.
Аноним 24/12/15 Чтв 19:24:56 #172 №321136 
Верно ли мое доказательство того, что множество алгебраических чисел счетно?

Пусть A_n - множество комплексных z таких, что z является корнем многочлена степени n с рациональными коэффициентами. Каждое z из A_n однозначно определяется коэффициентами многочлена и номером корня (от 1 до n). Поэтому |A_n| <= |Z^n x {1, ..., n}|, то есть A_n не более чем счетно.

Множество алгебраических чисел - это объединение всех A_n для n от 1 до бесконечности. Так как объединение счетного количества не более чем счетных множеств счетно (я верю в аксиому выбора), A счетно.
Аноним 24/12/15 Чтв 19:40:14 #173 №321151 
>>321121
Это классическая задача. Ответ: сумма (-1)^(k+1)/(k!), k от 1 до n и это стремится к e^-1.
Аноним 24/12/15 Чтв 19:43:19 #174 №321154 
>>321151
Только это я наоборот нашел вероятность того, что кому-то попадется свой подарок. Этот ряд, кстати, сходится очень быстро. При n = 10 сумма ряда отличается от e не более чем на 10^-7.
Аноним 24/12/15 Чтв 19:45:08 #175 №321157 
>>321136
Верно.
Аноним 24/12/15 Чтв 19:51:19 #176 №321159 
>>321136
>Так как объединение счетного количества не более чем счетных множеств счетно (я верю в аксиому выбора)
а где тут аксиома выбора-то?
Аноним 24/12/15 Чтв 20:47:46 #177 №321174 
14509792666270.png
>>321151
Все таки e^-1 это вероятность того, что никто не получит свой. В общем, вероятность того, что кто-нибудь получит свой подарок - это количество перестановок, в которых какая-нибудь цифра переходит сама в себя, поделить на количество всех перестановок. Количество таких перестановок можно найти по формуле включения-исключения и получится пикрелейтед.
Аноним 25/12/15 Птн 00:21:45 #178 №321213 
Есть кто-нибудь в треде, кто доволен своей жизнью? Я чет нихуя не доволен.
Аноним 25/12/15 Птн 00:34:46 #179 №321216 
>>321213
а чем занимаешься-то? И выхлоп есть какой?
Аноним 25/12/15 Птн 00:39:49 #180 №321218 
>>321213
вздернись лiл
Аноним 25/12/15 Птн 00:55:45 #181 №321222 
>>321213
Я тож ниоч, потому что каждый день всё больше понимаю насколько я плохо на самом деле знаю математику, насколько я отстаю от топчиков, задрачивавших курсы нму ещё со старших классов школы и от осознания того, что я никогда их уже не догоню.
Аноним 25/12/15 Птн 01:15:03 #182 №321223 
>>321136
интересно - множество действительных чисел несчетно. а алгебраических счетно. а алгебраические это расширение множества действительных на комплексную ось. сказать тебе где ты обосрался?
>Каждое z из A_n однозначно определяется коэффициентами многочлена и номером корня (от 1 до n).
контр пример - чесло 1. чесло 1 принадлежит к множеству z? по твоему доказательству следует что чесло 1 однозначно определяется коэффициентами многочлена и номером корня. многочлен x^n-1=0. как видишь неоднозначно. канает бесконечно много. вот поэтому то ты и доказал полную хуйню
Аноним 25/12/15 Птн 01:26:06 #183 №321224 
>>321223
Блядь, усыпите его.
Аноним 25/12/15 Птн 02:26:38 #184 №321230 
14509995985580.png
>>321101
Матаны, неужели вы меня подведете? А раньше всегда помогали и объясняли. Ну помогите тогда хотя бы доказать, что любая коммутативная группа 4 порядка изоморфна группе 4 поворотов (на 90 гралусов каждый) правильного четырехугольника вокруг центра.
Аноним 25/12/15 Птн 03:30:59 #185 №321238 
>>321230
>Ну помогите тогда хотя бы доказать, что любая коммутативная группа 4 порядка изоморфна
Это неправда. Та группа которую ты имеешь ввиду это циклическая группа Z_4 (только не четыре поворота по 90 градусов каждый лол, а четыре поворота на 90, 180, 270 и 0 градусов). А среди абелевых групп порядка четыре есть ещё и Z_2 x Z_2.
Аноним 25/12/15 Птн 04:08:51 #186 №321242 
>>321238
>только не четыре поворота по 90 градусов каждый лол, а четыре поворота на 90, 180, 270 и 0 градусов
Да! Я просто туповат слегка, не заметил, что хрень написал. Да и задача (у Александрова) формулируется про изоморфизм либо группе поворотов, либо клейновской группе. Это я тоже просохатил.

Поясни про то как это строго доказать, пожалуйста.
Аноним 25/12/15 Птн 04:21:51 #187 №321244 
>>320047 (OP)
>>318830
Прошло с сотку постов срача, взаимных обвинений в ватанстве, хуите про эллипсисы и так далее, но это ДУ так никто нормально и не решил:
>y"=2y^3
Аноним 25/12/15 Птн 04:31:51 #188 №321245 
>>321244
y(x) = _C2JacobiSN((ix+_C1)*_C2, i)
Аноним 25/12/15 Птн 05:26:26 #189 №321247 
>>321010
Категорией называется ориентированный граф, на рёбрах которого введены ассоциативная операция композиции и отношение равенства, причём в каждой вершине есть одна петля, нейтральная по этой операции. Вершины графа называются объектами, рёбра называются морфизмами (или стрелками).

Теория категорий использует аксиоматику NBG, в которой вдобавок к понятию множества есть понятие собственного класса. Собственный класс - это очень большая совокупность, которая не может являться множеством из-за своей величины. Например, класс всех множеств - это собственный класс. Совокупность всех объектов категории может являться не только множеством, но и собственным классом. Однако если A и B - два объекта категории, то совокупность всех морфизмов из A в B обязана являться множеством.

Любой класс математических объектов может быть смоделирован морфизмами какой-то категории (не объектами, а именно морфизмами). Например, мы можем зафиксировать какое-то поле K и рассмотреть категорию всех матриц над K. Объектами этой категории будут натуральные числа. Морфизмами между объектами m и n будут всевозможные матрицы из m строк и n столбцов. То есть вершинами графа являются натуральные числа; любая матрица размера m на n отождествляется с одним каким-то ребром из m в n. Другие примеры - категория всех множеств (объекты - множества, морфизмы - отображения), категория всех групп (объекты - группы, морфизмы - гомоморфизмы), категория всех топологических пространств (объекты - топологические пространства, морфизмы - топологические отображения), примеров очень много.

Теория категорий полезна в основном как язык.
Аноним 25/12/15 Птн 05:29:30 #190 №321248 
>>321159
Это одна из её формулировок, алё.
Точнее, конкретно это - формулировка аксиомы счётного выбора.
Аноним 25/12/15 Птн 11:04:43 #191 №321260 
>>321248
чёт ты гонишь маленько. Аксиома выбора говорит о том, что ты из каждого множества алгебраических чисел, являющихся корнями степени n, можешь выбрать по одному корню и снова будет множество.
Ты же используешь то обстоятельство, что объединение счётного числа не более чем счётных множеств не более чем счётно. Хз где тут аксиома выбора, разве что совсем неявно где-то.
Аноним 25/12/15 Птн 12:05:41 #192 №321267 
>>321247
> Категорией называется ориентированный граф, на рёбрах которого введены ассоциативная операция композиции и отношение равенства, причём в каждой вершине есть одна петля, нейтральная по этой операции. Вершины графа называются объектами, рёбра называются морфизмами
чето мне такое определение через граф не нравится
во-первых
> на рёбрах которого введены ассоциативная операция композиции
это не совсем бинарная операция, она определена только для ребер с общей вершиной направленных в одну сторону! т.е. fgh ассоциативно тогда когда данная композиция "имеет смысл"
во-вторых
> на рёбрах
тогда это не просто граф а мультиграф (если верить вики)!
между 2 вершинами в обычном ориентированном графе может быть только 1 ребро, у нас же в категорях между объектами может быть много морфизмов! т.е. у нас не просто ориентированный граф а что-то вроде мультиграфа
Аноним 25/12/15 Птн 12:58:17 #193 №321285 
>>321267
Это не просто мультиграф, это по большей части всегда граф с бесконечным количеством вершин, причём настолько большим что это даже иногда и не множество, и бесконечным количеством стрелок.
Аноним 25/12/15 Птн 13:40:53 #194 №321289 
14510400531670.png
14510400531691.png
>>321260
Это ты, уёбок, засрал весь тред своей мелкобуквенной тупостью? Съеби откуда пришел.
Аноним 25/12/15 Птн 14:00:14 #195 №321290 
14510412141670.png
Вот у нас есть функция, нарисованна белым, есть её производная, нарисованна синим, и тангенс угла зеленый.
Где между ними связь? Ведь производная функции равна тангенсу угла наклона касательной к функции в данной точке. Или я все слишком буквально принял?
Аноним 25/12/15 Птн 14:03:55 #196 №321293 
Спасибо за пояснения по категориям, анончики! В общих чертах понял о чем этот раздел. Может посоветуете книжку попроще про категории?
Аноним 25/12/15 Птн 14:04:26 #197 №321294 
>>321290
И да, как быть с касательной на промежутке от минус одного до одного. Ведь тогда касательная будет секущей.
Аноним 25/12/15 Птн 14:09:57 #198 №321295 
Здраствуйте, аноны. У меня есть вопрос насчёт бесконечности.
Правда ли что дроби или неполные числа (1.5, 0.37 и тому подобные) невозможно использовать в бесконечности как обычные числа? Если будет непонятно то я постараюсь объяснить.
Аноним 25/12/15 Птн 14:11:08 #199 №321296 
>>321295
Не понятно. Объясняй.
Аноним 25/12/15 Птн 14:16:05 #200 №321297 
>>321296
Хорошо. Вот представь бесконечность, состоящую из простых чисел, такие как 1,2,3,4 и так далее.
А теперь представь дроби. Сначала 1/1,1/2,1/3, и так далее. Потом тоже самое но с двойкой, то-есть 2/1,2/2,2/3. Но если первое бесконечное то как мы тогда имеем второе? Опять-таки, если непонятно объяснил то приведу другой пример.
Аноним 25/12/15 Птн 14:16:42 #201 №321298 
>>321242
Погугли теорему Лагранжа. Подумай какой порядок может иметь элемент в группе из четырёх элементов и рассмотри случаи.
Аноним 25/12/15 Птн 14:17:17 #202 №321299 
>>321293
Lawvere - Conceptual Mathematics
Аноним 25/12/15 Птн 14:21:21 #203 №321301 
>>321297
Числа не квантуются. Мы имеем бесконечное количество "кусочков" чисел между еденицей и двойкой. Точнее, мы можем поделить отрывой между еденицей и двоек на бесконечное количество кусочков. Обоснование этого лежит где то в глубинах математики, где отрыв от реальности, гроб гроб кладбище, удвоение объема тела после разрезания его на бесконечные кусочки.
Аноним 25/12/15 Птн 14:22:12 #204 №321302 
>>321301
и складывания обратно*
Аноним 25/12/15 Птн 14:24:32 #205 №321304 
>>321302
Спасибо за пояснение, а то я родителям уже все мозги запудрил. Я ещё хотел спросить что-то похожее, но думаю не стоит ведь твой ответ подходит и туда.
Аноним 25/12/15 Птн 14:25:56 #206 №321305 
>>321267
Бинарная операция не обязана быть вполне определённой. В нормальных учебниках, и даже у Бурбаки, всегда рассматривают частично определённые бинарные операции.

Под графом же в математике, ещё со времён Эйлера и его задачи о мостах, понимается именно "мультиграф". Строгое определение графа есть у Маклейна.

>>321260
Докажем, что объединение счётного семейства счётных множеств {xij} счётно.
Расположим их в виде бесконечной таблицы
x11 x12 x13 ...
x21 x22 x23 ...
x31 x32 x33 ...
...

Затем известным приёмом, "змейкой", расположим их в виде строки:
M = x11 x12 x21 x31 x22 x13 ...

Казалось бы, всё хорошо, получили счётное множество. Но кто сказал, что совокупность M является множеством?
Аноним 25/12/15 Птн 14:30:26 #207 №321306 
>>321290
>>321294
Ну бамп, че вы меня игнорируете а графы обсуждаете? Кубики рубика собираете или овцу, капусту и волка через реку перевозите?
Аноним 25/12/15 Птн 14:31:41 #208 №321307 
>>321306
Кстати кубик Рубика не очень то и сложный механизм.
Аноним 25/12/15 Птн 14:33:26 #209 №321308 
>>321306
Колмогоров, "Алгебра и начала анализа". Прочитай.
>есть её производная
Производная функции f в точке x - это число. В школьном случае равное тангенсу угла наклона касательной, проведённой к графику функции в этой точке.
Производная функция функции f - это функция, которая каждой точке x сопоставляет производную функции f в точке x.

"Производная функции в точке" и "производная функция функции" - два разных понятия.
Аноним 25/12/15 Птн 14:35:18 #210 №321309 
>>321307
Он по графам решается.
Аноним 25/12/15 Птн 14:37:46 #211 №321311 
>>321309
Я не знал если честно, но я знаю что для решения этой головоломки применяются алгоритмы. Можешь показать какие графы, пожалуйста? Очень хотелось бы посмотреть.
Аноним 25/12/15 Птн 14:39:21 #212 №321312 
>>321308
>функция, которая каждой точке x сопоставляет производную функции f в точке x.
Я знаю что сопоставляет через пределы/бесконечно малые. Но где между ними связь? Или только через первородную?
>В школьном случае равное тангенсу угла наклона касательной, проведённой к графику функции в этой точке.
Почему тогда график тангенса не совпадает с графиком производной? Ну и
> как быть с касательной на промежутке от минус одного до одного. Ведь тогда касательная будет секущей.
Аноним 25/12/15 Птн 14:42:24 #213 №321313 
>>321312
>Колмогоров, "Алгебра и начала анализа". Прочитай.
Аноним 25/12/15 Птн 14:44:06 #214 №321314 
>>321313
Я читаю зельдовича. Там все и так объясняется, но я увидеть не могу этой связи тангенса с производной на графике. В твоем комогорове это объясняется или сразу же что то уровня пикчи надо делать?
Аноним 25/12/15 Птн 14:44:27 #215 №321315 
14510438675200.jpg
>>321314
>>321313
Аноним 25/12/15 Птн 14:55:31 #216 №321320 
Аноны, а что вы можете сказать насчёт Апорий Зенона?
Аноним 25/12/15 Птн 14:57:05 #217 №321321 
>>321320
Логические парадоксы.
Ошибка в изначально ложных посылках логики и отсутствия понятий времени/расстояния/т.д.
Аноним 25/12/15 Птн 14:59:48 #218 №321322 
14510447882350.png
>>321223
Заскринил. Люблю свою коллекцию даунов.
Аноним 25/12/15 Птн 15:03:20 #219 №321323 
>>321321
То-есть если я приведу один из парадоксов своему учителю по математике, то он посмотрит на меня как на идиота? Я думал что он просто совмещает математику где она по сути не нужна (Например Ахиллес и черепаха)
Аноним 25/12/15 Птн 15:05:40 #220 №321324 
>>321320
Апории Зенона демонстрируют, что понятия времени, делимости, скорости, движения и предельного перехода не являются интуитивно ясными и нуждаются в строгом определении.

>>321323
Иди к ЕГЭ готовься, что ты на анонимном сайте для маргиналов-извращенцев забыл?
Аноним 25/12/15 Птн 15:06:17 #221 №321325 
>>321323
Значит возьми черепаху и попробуй её обогнать. Проведи опыт, сформулируй теорию, объяви о своих результатах и предложи другим проверить твои результаты.
Аноним 25/12/15 Птн 15:07:43 #222 №321326 
>>321324
Вообще-то я эти два поста запостил...
Аноним 25/12/15 Птн 15:08:25 #223 №321327 
>>321325
Я бы очень рад, но я в математике немножечко туп.
Аноним 25/12/15 Птн 15:10:04 #224 №321329 
>>321311
Кубик рубика сам по себе это интересный пример, чтобы использовать теорию групп.
http://www.math.harvard.edu/~jjchen/docs/Group%20Theory%20and%20the%20Rubik%27s%20Cube.pdf
Аноним 25/12/15 Птн 15:12:10 #225 №321330 
>>321329
Спасибо за ссылку, анон.
Аноним 25/12/15 Птн 15:18:25 #226 №321332 
14510459057400.png
>>321322
Вбрось.
Аноним 25/12/15 Птн 15:22:37 #227 №321334 
14510461572420.png
>>321332
Это мой пост у тебя на скрине. Хуй тебе, а не моя коллекция.
Аноним 25/12/15 Птн 15:29:50 #228 №321335 
>>321223
> контр пример
значение знаешь?
Аноним 25/12/15 Птн 15:29:57 #229 №321336 
14510465973730.webm
>>321334
Любишь черезчур абстрактной наукой заниматься? Пятиклассников - то высшим категориям научил уже?
Аноним 25/12/15 Птн 15:32:40 #230 №321338 
14510467600610.png
>>321334
>Это мой пост
Аноним 25/12/15 Птн 15:33:51 #231 №321339 
>>321290
ты просто всё в кучу смешал и поэтому нихера и не понимаешь. давай зайдем с другой стороны, а именно с определения производной через ебучий пердел. производная это же по сути предел отношения разности игрека к разности икса при стремлении разности икса к нулю. что происходит другими словами. есть у тебя график функции y=x^2. найдем для него производную в точке 2. для этого возьмем интервал х1=1.9 х2=2.1. соответственно y1=3.61 y2=4.41, x2-x1=0.2, y2-y1=0.8. а производная будет y2-y1/x2-x1=0.8/0.2 = 4. теперь че мы получили графически - из точек x1 и x2 поднимем перпендикуляры к нашей параболе,а там где они пересекут параболу отбросим перпендикуляры на y. нетрудно заметить что этим мы отсекли какой то небольшой кусочек параболы. не точку, а отрезок, причем криволинейный. а еще мы получили квадратик, у которого стороны будут y2-y1 и х2-x1, а вот этот кусочек параболы будет выступать эдакой кривоватой диагональю. или можно рассмотреть треугольник, у которого гипотинузой будет этот самый криволинейный кусок параболы. но катеты этого треугольника будут y2-y1 и x2-x1. мусье согласен?
теперь в прямоугольном треугольнике, как должно быть известно мусье, тангенс, по определению это отношение противолежашего катета к прилежашему. тоесть y2-y1/x2-x1. вроде всё пока ясно но не до конца. на самом деле, по определению производной x2-x1 должен стремится к нулю, т.е. в идеале мы должны взять такой маленький x2-x1, при котором выделенный кусочек параболы будет уже не кривым а малюсенькой прямой. или точкой. но приближать так можно бесконечно долго. и приближая x2-x1 к нулю мы будем делать этот наш треугольничег бесконечно маааленьким, гипотинузка будет становится прямой и в какоой то момент приближения наша посчитанная производная будет стремится к какому то числу. к какому? по формуле производных к числу 2x. т.е. если в точке 2 то к числу 4. Теперь я надеюсь ты понял какой тангенс считает производная и никакой связи с графиком тангенса она не имеет. про касательную - касательная к параболе в точке 2 будет ведь прямой линией?. при этом, эта прямая линия будет под таким же углом к катету x2-x1 нашего мини треугольника, и вообще, повторять нашу минигипотинузку. так сказать накладыватся на неё. но x2-x1 паралелен оси x, поэтому этот несчастный угол между нашей гипотинузкой и катетом x2-x1 равен углу между касательной и осью x. углы равны? равны и их тангенсы. поэтому и говорят что производная функции в точке равна углу наклона касательной к графику в этой точке. для некоторых функций можно вывести некие функции, которые будут сопоставлять каждой точке графика изначальной функции значение угла касательной в этой точке. например для x^n производной будет n*x^n-1. т.е. используя эту функцию можно не заморачиваясь с пределами найти этот тангенс для каждой точки функции.
Аноним 25/12/15 Птн 15:35:12 #232 №321340 
14510469128000.webm
у миня тоже есть чел с доской
Аноним 25/12/15 Птн 15:54:45 #233 №321346 
>>321336
И эту вебмку я нарезал.
Аноним 25/12/15 Птн 16:11:46 #234 №321354 
14510491061720.png
>>321230
Кстати, твой вопрос сделал меня вспомнить вопросы, над которыми я когда-то думал, но по-молодости ничего не придумал и забыл.
Вброшу, раз тут, раз тухловато в качестве исследовательской задачки:
1) Какие конечные (ну и до кучи, бесконечные)
группы можно реализовать как группы всех симметрий (аффинных преобразований/изометрий) каких-то подмножеств на плоскости (или в пространстве)?
2) Какие условия на множество отвечают тому, что группа его симметрий будет конечной или бесконечной? Например, у диска симметрий бесконечное количество, а у многоугольника конечное, почему? Какие ещё множества с бесконечными симметриями есть?
3) Если мы приближаем диск многоугольниками (ну или вообще, множество другими множествами) какой операции это соответствует на уровне групп симметрий?
Аноним 25/12/15 Птн 16:30:15 #235 №321358 
>>321339
Это я и так знал. До этого момента
>но приближать так можно бесконечно долго. и приближая x2-x1 к нулю мы будем делать этот наш треугольничег бесконечно маааленьким, гипотинузка будет становится прямой и в какоой то момент приближения наша посчитанная производная будет стремится к какому то числу. к какому? по формуле производных к числу 2x. т.е. если в точке 2 то к числу 4. Теперь я надеюсь ты понял какой тангенс считает производная и никакой связи с графиком тангенса она не имеет.

Сразу надо было про треугольник написать, я вроде не настолько даун.
Аноним 25/12/15 Птн 16:36:49 #236 №321359 
>>321358
ну прости - откуда мне знать насколько ты даун - пытался обьяснить как самому себе в школе. не помню уже насколько я был дауном тогда. но помню что этот вопрос меня ебал так же как и сейчас тебя. и конечно я не помню знал я тогда про треугольник или не знал
Аноним 25/12/15 Птн 16:37:59 #237 №321360 
>>321354
1 - первая и вторая группа движений, если сортировать по сохранению/не сохранению порядка обхода
Аноним 25/12/15 Птн 16:39:21 #238 №321361 
>>321358
если не знал про треугольник - не то что настолько, но еще болеей
Аноним 25/12/15 Птн 17:17:00 #239 №321367 
>>321360
Что-то не соображу, а у какого множества циклическая группа даёт ВСЕ его симметрии?
Аноним 25/12/15 Птн 18:00:47 #240 №321377 
14510556478340.jpg
>>321340
>>321338
>>321336
>>321334
Ковры на стенах - наши внутренние леса,
А доски - чёрные озёра, серебряным блеском отражающие лунный свет.
Аноним 25/12/15 Птн 18:44:55 #241 №321389 
14510582955000.png
14510582955001.png
Можете подсказать, в каком направлении двигаться, чтобы разложить прикреплённый многочлен на множители традиционными способами?
На втором пике результант моих максимальных трёхчасовых усилий, не знаю, что тут делать дальше с этим кубом и в верном ли направлении я пошёл.
Аноним 25/12/15 Птн 18:54:19 #242 №321396 
>>321389
Как бывалый скажу, если что то не становится проще, значит делаешь что то не так.
Аноним 25/12/15 Птн 19:23:38 #243 №321400 
>>321389
тут можно поискать корни среди целых делителей свободного члена (-12). Как найдешь - подели многочлен на (a-r), где r - любой найденный корень. И повторяй до успеха.
Аноним 25/12/15 Птн 19:44:08 #244 №321409 
>>321298
>>321299
Спасибо!
Аноним 25/12/15 Птн 22:07:40 #245 №321433 
>>321389
Разложи на произведение квадратичных многочленов с неопределёнными коэффициентами, раскрой скобки и приравняй к коэффициенты. Получится, конечно, системка из квадратичных уравнений, но её уже можно позарачивать.
Аноним 25/12/15 Птн 23:51:38 #246 №321449 
>>321396
>>321400
>>321433
Спасибо, аноны! Я в экстазе, ведь у меня в итоге всё получилось.
Аноним 26/12/15 Суб 00:55:36 #247 №321458 
1. Почему минус на минус - плюс?
2. Вербит или кто-то типа него как-то где-то сказал, что первый курс сильнее, мотивированнее и выносливее(способнее многое и больее переварить?) второго(и третьего? и далее?). Пояснил тем, что мотивации и свежести больше. Дайте ссылку, в общем.

Вот сейчас я второкурсник и я жто почувствовал. Как же заебался.

Аноним 26/12/15 Суб 11:09:46 #248 №321506 
14511173869640.png
Народ, помогите пожалуйста решить уравнение В
Аноним 26/12/15 Суб 11:09:58 #249 №321507 
>>321458
> Почему минус на минус - плюс?
Если ты спрашиваешь, почему -(-a) = a, то по определению аддитивной инверсии.

Если ты спрашиваешь, почему (-x)(-y) = xy, то сначала надо доказать, что -a = (-1)a, и получим (-x)(-y) = (-1)(x)(-1)(y) = (-1)(-1)xy = xy, где второе равенство следует из коммутативности умножения, а третье из того, что мультипликативная инверсия (-1) это (-1).

Доказательство того, что -a = (-1)a:
(-1)a = -a + a + (-1)a = -a + (1 - 1)a = -a + 0a = -a
Аноним 26/12/15 Суб 11:19:29 #250 №321508 
>>321458
>на 5м курсе будешь думать что ваще бог и будешь всегда заебаным. по крайней мере будешь так думать. и будет лениво что либо делать. а потом начнешь серьезно рабротать и будешь с умилением вспоминать эти деньки
Аноним 26/12/15 Суб 11:51:49 #251 №321510 
реквестирую годных задач уровня imc порешать на матан и теорию чисел
Аноним 26/12/15 Суб 13:22:08 #252 №321522 
>>321507
> а третье из того, что мультипликативная инверсия (-1) это (-1).
Чет я проебался. Вот это хз как доказать.
Аноним 26/12/15 Суб 13:45:19 #253 №321533 
>>321507
>>321522
Докажем, что (-a)-1 = -(a-1).
(-a)^-1 = a-1a(-a)-1 = a-1(-1)(-a)(-a)-1 = a-1(-1) = -(a-1).

Теперь
(-1)-1 = -(1-1) = -1.
Отсюда
(-1)(-1) = 1.
Аноним 26/12/15 Суб 13:57:28 #254 №321539 
>>320047 (OP)
Двачеты, помогите. Есть быстрое и простое доказательство существования первообразного корня по модулям p^a и 2p^a? У Нестеренко хуита с трехэтажными степенями, у Виноградова лучше, но все равно плохо. через полтора часа зачет по теории чисел, а я эту хуиту не знаю как доказать.
Аноним 26/12/15 Суб 13:58:03 #255 №321540 
>>321533
>Докажем, что (-a)-1 = -(a-1).
ясно
>Теперь
понятно
>Отсюда
но почему?
Аноним 26/12/15 Суб 14:01:39 #256 №321543 
>>321540
МОТЕМАТИКИСКОЗАЛЯ, РЯЯЯЯ
И это не шутка, а реальное доказательство.
Аноним 26/12/15 Суб 14:25:09 #257 №321552 
14511291092900.png
>>321539
Хелп плз. Пик не осилю.
Аноним 26/12/15 Суб 14:35:10 #258 №321554 
14511297102430.png
Почему считается, что значение 1 в степени inf не определено, а не равно 1?
Аноним 26/12/15 Суб 14:37:20 #259 №321555 
14511298404330.png
>>321554
подумай.
Аноним 26/12/15 Суб 14:43:23 #260 №321557 
>>321554
Бесконечность не определена в общепринятых арифметических полях.
Аноним 26/12/15 Суб 14:48:47 #261 №321562 
>>321540
Потому что
(-1)(-1) = (-1)(-1)-1 = 1.
Аноним 26/12/15 Суб 15:07:01 #262 №321565 
>>321557
Математика - символьная наука?
Аноним 26/12/15 Суб 18:06:59 #263 №321579 
>>321557
Вообще-то расширенная вещественная прямая является общепринятой конструкцией, хотя и не является ни полем, ни даже кольцом. Бесконечность апеллирует к топологическим свойствам R, и о расширенной прямой правильнее думать как о топологическом пространстве.
Аноним 26/12/15 Суб 18:07:05 #264 №321580 
14511424254080.jpg
Три измерения, аноны.
Направьте, как доказать, что пересечение сфер - окружность.
Аноним 26/12/15 Суб 18:32:53 #265 №321583 
>>321580
Очевидно, что пересечение сфер - окружность. Что и требовалось доказать.
Аноним 26/12/15 Суб 19:06:28 #266 №321590 
>>321580
Любая секущая через сферу дает окружность.
Аноним 26/12/15 Суб 19:22:06 #267 №321592 
>>321590
Нет, пересечение сферы и вытянутого сферойда даст овал.
Аноним 26/12/15 Суб 19:22:42 #268 №321594 
>>321592
Или две окружности
Аноним 26/12/15 Суб 19:23:18 #269 №321595 
>>321594
или два овала
Аноним 26/12/15 Суб 20:06:16 #270 №321605 
14511495760400.jpg
14511495760421.png
14511495760422.png
Сайч, поясни пожалуйста, будь любезен, куда обратиться за умом-разумом по моей проблеме, или хотя бы пните в нужную сторону.
Пишет тебе безвестный кодер, перед которым стоит задача тригонометрического сорта: есть одна трапеция в двумерном пространстве, тупой угол которой нужно скруглить, пришив эллипс без изломов (т.е. касательные эллипса должны быть идентичны векторам-направляющим отрезков трапеции).
Таким образом, мне нужно получить точки центра эллипса, его A и B по двум точкам и двум касательным. Оси эллипса, естественно, параллельны соответствующим осям координат.
Проблема оказалась для меня сложнее, чем я ожидал. Заранее спасибо за любую инфу.
Аноним 26/12/15 Суб 20:13:52 #271 №321609 
>>321580
Полуштангой будем называть окружность и прямую, проходящую через центр оной окружности перпендикулярно плоскости оной окружности. Окружность будем называть диском, прямую будем называть грифом.

Семейство полуштанг будем называть концентрическим, если они все имеют один и тот же гриф, и у любых двух разных дисков центры не равны (игрушка "ёлочка"; неравенство центров означает, что никакие два диска не нанизаны на гриф в одном и том же месте).

Ясно, что диски концентрического семейства полуштанг либо равны, либо не пересекаются. Это означает, что если точка принадлежит объединению дисков концентрического семейства полуштанг, то она принадлежит лишь одному диску из семейства.

Пусть есть концентрическое семейство полуштанг такое, что их гриф является направленной прямой. Тогда мы будем называть это семейство убывающим, если радиусы окружностей убывают при движении вперёд по прямой.

Пусть есть два убывающих семейства полуштанг таких, что их диски расположены в параллельных плоскостях, а их грифы равны, но различаются ориентацией. То есть на один и тот же гриф нанизаны два семейства параллельных дисков так, что при движении по прямой вперёд радиусы дисков первого семейства возрастают, а радиусы дисков второго семейства убывают, и наоборот при движении назад.

Рассмотрим пересечение M объединения A дисков первого семейства и объединения B дисков второго семейства. Предположим, что это пересечение непусто. Тогда каждая точка x из M принадлежит как A, так и B. Тогда x принадлежит одному какому-то диску Ax первого семейства и одному какому-то диску Bx второго семейства. Ax и Bx расположены в параллельных плоскостях, а их центры лежат на прямой, перпендикулярной их плоскостям. Ax и Bx могут пересекаться тогда и только тогда, когда Ax = Bx. Таким образом, каждая точка M входит в M вместе со всем диском, то есть M есть подмножество дисков обоих семейств полуштанг.

Так как M непусто, M содержит по крайней мере один диск полуштанги. Ясно, что M не может содержать больше одного диска, так как при движении по грифу вперёд радиусы дисков из M должны и возрастать, и убывать, что абсурдно. Поэтому пересечение A и B, коль скоро оно непусто, есть окружность.

Если мы возьмём две неравные, но пересекающиеся полусферы, соединим их центры прямой и опустим на эту прямую пучок перпендикулярных плоскостей, то получим как раз рассмотренные выше два семейства полуштанг (ибо пересечение сферы и прямой - окружность). Поэтому две пересекающиеся неравные полусферы пересекаются только по окружности.

Отсюда можно вывести и утверждение для сфер.
Аноним 26/12/15 Суб 20:16:14 #272 №321610 
>>321605
Я вроде сейчас надумал, что центр эллипса - пересечение перпендикуляров к v1 и v2, после этого я смогу перенести p1 и p2 в локальные координаты эллипса - а там всё должно разрешиться стандартным способом. Это кратчайший путь, достойный блаародного джентльмена?
Аноним 26/12/15 Суб 20:20:28 #273 №321611 
14511504282320.png
>>321605
Скруглять можно окружностями разных радиусов. Радиус скругления известен?
Аноним 26/12/15 Суб 20:22:01 #274 №321612 
>>321611
Нужен эллипс. По идее он будет единственным, или я ошибаюсь?
Нужно было на картинке яснее показать, что иском именно эллипс, прошу прощения.
Аноним 26/12/15 Суб 20:25:26 #275 №321614 
>>321612
Ну вот же картинка >>321611, показывающая неединственность скругления. Или уточни условие.
Аноним 26/12/15 Суб 20:29:36 #276 №321616 
>>321609
блять, охуеть. А если так:
Множество точек, образованных пересечением двух сфер есть пересечение множеств точек первой и второй сферы, а по определению это точки, удалённые от C_1 на R_1 И от C_2 на R_2, где C - центры сфер, R - их радиусы.
Потом записать систему:
(x_т-x_s1)^2 + (y_т - y_s1)^2 + (z_т - z_s1)^2 = r_1^2 ,
(x_т-x_s2)^2 + (y_т - y_s2)^2 + (z_т - z_s2)^2 = r_2^2
и невероятным манёвром привести её к виду уравнения окружности в трёхмерном пространстве?
Аноним 26/12/15 Суб 20:29:49 #277 №321617 
14511509892440.png
>>321614
Ну по идее даже с окружностями множественность не прокатит, так как есть условие соответствия касательным векторам.
Аноним 26/12/15 Суб 20:32:10 #278 №321618 
>>321616
>уравнения
Не наглядно.
Аноним 26/12/15 Суб 20:36:45 #279 №321620 
>>321592
Пруфни что ли.
Аноним 26/12/15 Суб 20:40:12 #280 №321622 
>>321618
важен результат. Его же можно получить таким методом?
Аноним 26/12/15 Суб 20:42:47 #281 №321625 
>>321622
Результат получен, что ещё нужно.
Аноним 26/12/15 Суб 20:44:48 #282 №321627 
>>321625
Я не вижу тут результата. Куда смотреть?
Аноним 26/12/15 Суб 21:01:52 #283 №321633 
>>321605
>>321617
Вопрос снимается, решил запилить Безье и не ебать мозг.
Аноним 26/12/15 Суб 21:31:48 #284 №321634 
14511547081050.png
Матаны, не могу решить задачку по Ан. гему. Подскажите, что да как. Был бы очень благодарен.
Аноним 26/12/15 Суб 22:46:32 #285 №321644 
>>321634
Выражаешь икс через одно уравнение и подставляешь его в другое на место икса. Находишь икс. Ставишь значение икса в уравнения. С игреко делаешь то же самое.
Аноним 27/12/15 Вск 03:05:56 #286 №321704 
>>321627
Выше.
Аноним 27/12/15 Вск 05:13:49 #287 №321712 
Матаны, дебил, стремящийся к знаниям, но не знающий в математике почти ничего, снова выходит на связь! Посоветуйте, пожалуйста, видеолекций (хочу обмазываться параллельно с чтением учебников) по общей алгебре. Желательно уровня ньюфагов. Если будут на русском, то вообще отлично (а то немного очкую, что с иноязычной терминологией все в голове перемешается).
Аноним 27/12/15 Вск 08:57:12 #288 №321721 
>>321712
http://www.mathnet.ru/php/presentation.phtml?option_lang=rus
>Лекции и семинары Независимого Московского университета
Аноним 27/12/15 Вск 16:19:08 #289 №321767 
Как понятными словами сказать, что такое предел функции?
Аноним 27/12/15 Вск 16:48:40 #290 №321783 
>>321712
https://www.lektorium.tv/course/26552
Аноним 27/12/15 Вск 18:36:17 #291 №321810 
>>321580
>>321616
Пусть окружности имеют центры O_1, O_2 и радиусы r_1, r_2. Возьмем начало координат в точке O_1 и возьмем ось X по направлению вектора O_1O_2, то есть O_1 имеет координаты (0, 0, 0), а O_2 - (t, 0, 0). Получаем систему уравнений
x^2 + y^2 + z^2 = r_1^2
(x - t)^2 + y^2 + z^2 = r_2^2
Отсюда
x = (r_1^2 - r_2^2 - t^2) / (2t)
и
y^2 + z^2 = r_1^2 - x^2.
То есть y^2 + z^2 равно некоторой константе (если она отрицательная, то сферы не пересекаются). Сферы пересекаются по окружности, которая лежит на оси X.

>>321609
> полуштанга
> гриф
> игрушка "елочка"
> коль скоро
Как же я проиграл с этих слов.
Аноним 27/12/15 Вск 18:43:21 #292 №321811 
>>321810
>Как же я проиграл с этих слов.
зато понятно
>То есть y^2 + z^2 равно некоторой константе
чего я и хотел. Няши в сай только отвлекают, так что просто добра тебе.
Аноним 27/12/15 Вск 19:37:30 #293 №321845 
>>321811
>чего я и хотел. Няши в сай только отвлекают, так что просто добра тебе.
Добавлю что важно ещё что и x равно константе - т.е. все такие точки (y,z) лежат в одной плоскости.
Аноним 27/12/15 Вск 19:41:09 #294 №321846 
>>321845
Я это сам уже понял, спасибо
Аноним 27/12/15 Вск 19:44:01 #295 №321847 
>>321810
> по окружности, которая лежит на оси X.
фикс:
которая лежит в плоскости, перпендикулярной оси X
Аноним 27/12/15 Вск 20:24:47 #296 №321851 
>>321721
Мне один знакомый говорил, что этот университет - худшая шарага в Москве. Типа там все за деньги покупается, и никто даже этого не скрывает. Или это не так? В любом случае, спасибо.
>>321783
Спасибо.

Кстати, на ютьюбе еще видел распиаренные лекции Романа Михайлова по теории групп. Как они? Стоит ли пробовать?
Аноним 27/12/15 Вск 20:59:43 #297 №321860 
>>321851
лал
Аноним 27/12/15 Вск 21:33:15 #298 №321864 
14512411958710.png
Посоны, я тут правильно раскрыл? надеюсь это не картофан
Аноним 27/12/15 Вск 22:10:27 #299 №321867 
Как доказать, что ax+bx=cx не имеет решений про x>2?
Аноним 27/12/15 Вск 22:24:10 #300 №321872 
>>321864
В третьей строке ошибка. Там должно быть
∀x(¬P (x) ∨ ¬Q(x, y))
Пример:
¬(∃x(x четное ∧ x нечетное)) ≡ ∀x(¬(x четное) ∨ ¬(x нечетное))
а не
(∀x ¬(x четное)) ∨ (∀x ¬(x нечетное))
Аноним 27/12/15 Вск 22:28:26 #301 №321873 
>>321851
>Типа там все за деньги покупается
лол
Аноним 27/12/15 Вск 22:59:03 #302 №321885 
>>321872
Это разве не одно и то же? Или так нельзя раскрывать скобки?
Аноним 27/12/15 Вск 23:04:32 #303 №321888 
>>321867
Поздно схватился. Ферма уже доказал.
Аноним 27/12/15 Вск 23:11:34 #304 №321892 
>>321885
Ыы почувствовал так, ты прав.
Аноним 27/12/15 Вск 23:48:27 #305 №321904 
>>321888
> Ферма уже доказал.
У него бумага кончилась и он не смог записать доказательство.
Аноним 27/12/15 Вск 23:55:15 #306 №321905 
Чувачки, помогите разобраться:
по таблице критических точек распределения Стьюдента нужно найти t с индексом "кр", известно только "k=n-2=19".
Ссылка на таблицу:
http://www.natalymath.ru/krit_student.html
Аноним 27/12/15 Вск 23:59:02 #307 №321906 
>>321872
А в итоге все равно то же самое получается?
Аноним 28/12/15 Пнд 01:18:03 #308 №321914 
>>321851
> Стоит ли пробовать?
Если ты "дебил, стремящийся к знаниям, но не знающий в математике почти ничего" то ты вряд ли что-то поймёшь кроме кулсторей про трипы и метафизику. А НМУ норм.
Аноним 28/12/15 Пнд 14:08:20 #309 №321968 
Сап, братва, ёба. Надо разобраться в том, что такое проективная плоскость, проективное пространство, проективная кривая/прямая, проективное замыкание. Посоветуйте литературу для новичков.
Аноним 28/12/15 Пнд 14:43:04 #310 №321972 
Может ли быть несчетное множество отдельных точек?
Аноним 28/12/15 Пнд 15:17:17 #311 №321977 
Почему любая аффинная плоскость состоит из по крайней мере четырех различных точек? Почему недостаточно трех точек?
Аноним 28/12/15 Пнд 15:21:24 #312 №321978 
>>321972
например если взять каждое действительное чесло, и сопоставить ему точку на числовой прямой получишь бесконечное количество точек. почему бесконечное? ну например 1 и 2. две точки. между ними есть 1,2 и 1,3. а между ними 1.22 и 1.29 и так до бесконечности
Аноним 28/12/15 Пнд 15:24:54 #313 №321979 
>>321977
Сам разобрался. Четвертая точка - точка пересечения двух прямых.
Аноним 28/12/15 Пнд 15:26:11 #314 №321980 
>>321978
Получишь множество рациональных точек.
Аноним 28/12/15 Пнд 15:35:09 #315 №321982 
>>321980
и чем оно не понраву? алсо не обязательно рациональных, могут затесаться так же и те, которые не будут представимы в виде q/n
Аноним 28/12/15 Пнд 15:47:51 #316 №321984 
>>321972
Может, например R^n с дискретной топологией или несвязное обьёдинение несчётного семейства множеств из тех же R^n с обычной топологией, в пространствах функций всяких. Вот в простом R^n с обычной топологией вроде не может, как доказать - не знаю.
Аноним 28/12/15 Пнд 15:48:17 #317 №321985 
>>321982
Оно счётно. Вопрос был про несчётное бесконечное множество изолированных точек.
Аноним 28/12/15 Пнд 16:21:16 #318 №321989 
>>321985
счетно не значит конечно, это раз. два - в вопросе не было оговорено про его счетность. в третьих ну возьми какое нибудь несчетное множество за основу вместо действительных чисел, раз не нравится счетность. сам нагуглишь несчетное?
Аноним 28/12/15 Пнд 16:27:16 #319 №321990 
>>321972
первая ссылка в гугле
http://math.stackexchange.com/questions/1443010/can-we-have-an-uncountable-number-of-isolated-points
Аноним 28/12/15 Пнд 16:27:39 #320 №321991 
>>321972
Отдельная точка - это точка x топологического пространства такая, что {x} - открытое множество?
Аноним 28/12/15 Пнд 16:29:45 #321 №321992 
>>321991
Насколько я понял, это он так изолированную точку называет.
Аноним 28/12/15 Пнд 17:56:26 #322 №322005 
>>321989
Ну, не оче выходит, 1/10 где-то. Завязывай.
Аноним 28/12/15 Пнд 19:24:01 #323 №322018 
>>321767
бамп
Аноним 28/12/15 Пнд 21:05:04 #324 №322034 
>>321767
это когда выполняется условие - если одна последовательность стремится к чему то(Xn->E), а другая, которая от неё зависит функционально(Y=f(X)), стремится к примеру к А (Yn->A), тогда говорят что у функции f(x),при х->E, существует предел,равный A. для более четкого понимания надо копать предел последовательности. там определяли какие то последовательности, а их чередование рассматривали как возрастание натуральных чисел. типа 1\х при стремлении n к бесконечности стремится к нулю. а предел функции это расширение понятия предел последовательности на более общий случай, когда сами n могут задаватся от пизды, а не строго по возрастанию. и тип говорят, что если последовательность,с помощью которой задают чередование стремится к чему то, а задаваемая при этом тоже стремится к чему то, то типа тогда вся эта ситуация - предел функции, лiл
Аноним 28/12/15 Пнд 21:46:33 #325 №322038 
>>322034
Но предел - это какая-то сущность или это просто словесное выражение, оборот речи?
Аноним 28/12/15 Пнд 23:30:53 #326 №322050 
14513346535000.jpg
Двач, вкачусь в этот тред с платиновым вопросом: как учить математику человеку, который от нее чуть не засыпает. Нужно по работе (наука). Тригонометрию в школе не учил, равно как и интегралы, пределы. Двач, выручай.
Аноним 28/12/15 Пнд 23:31:46 #327 №322051 
>>322050
Какая такая наука?
Аноним 28/12/15 Пнд 23:32:47 #328 №322052 
>>322050
>который от нее чуть не засыпает
Ты засыпаешь потому, что подсознательно считаешь математику ненужной, скучной и унылой. Подчини себе своё подсознание. Научись жадно хавать математику.
Аноним 28/12/15 Пнд 23:36:29 #329 №322053 
>>322038
блядство! почему ты не смотрел определение предела последовательности?!? там дается ответ на этот вопрос по ебаному хардококору!
>Пределом последовательности называют объект, к которому члены последовательности в некотором смысле стремятся или приближаются с ростом номера.
>объект
https://ru.wikipedia.org/wiki/%D0%9F%D1%80%D0%B5%D0%B4%D0%B5%D0%BB_(%D0%BC%D0%B0%D1%82%D0%B5%D0%BC%D0%B0%D1%82%D0%B8%D0%BA%D0%B0)
Аноним 28/12/15 Пнд 23:49:38 #330 №322055 
В треде есть профессиональные математики? Мне хотелось бы знать как трудно устроится и что для этого нужно.
Аноним 29/12/15 Втр 00:16:14 #331 №322058 
>>322055
Нужен военник и справка 086у.
Аноним 29/12/15 Втр 09:42:08 #332 №322107 
>>322058
где достать? мож купить проще?
Аноним 29/12/15 Втр 10:22:18 #333 №322110 
Знающий анон, скажи, а какие темы must have в общей топологии? Что нужно знать, чтобы не быть лаптем?
Аноним 29/12/15 Втр 11:01:38 #334 №322112 
14513760985640.png
>>320047 (OP)
Бля чуваки, нужен учебник из формата "азы математики для даунов" с большим количеством примеров, где все разжевано, который плавно выведет на комплексные числа, дифференцирование, интегрирование и прочий пиздец инста. Обычные учебники не канают т.к. предполагается наличие препода, который будет объяснять сложные, непонятные моменты.

Кратко суть, сильно захотел упороться схемотехникой, а там сплошные математические выкладки на по темы без понимания которых и сраный конденсатор не просчитать, я в ахуе. Школьные знания давно забыты и проебаны.

Спасибо.
Аноним 29/12/15 Втр 12:41:12 #335 №322118 
Где можно найти "Лекции по алгебраической геометрии" Ф. Севери в виде книжки? Для фонового чтения при изучении кодов, ассоциированных с многообразиями, советовали именно эти лекции в 60-х годах, ололо
Аноним 29/12/15 Втр 13:03:20 #336 №322119 
>>322112
> комплексные числа
гиперкомплексные числа кантор солодовников
даже если прочтешь только начало то заодно еще освоишь кватернионы и азы векторного исчисления
вот это вот еще
ilib.mccme.ru/pdf/ponarin.pdf
Аноним 29/12/15 Втр 13:15:37 #337 №322121 
Почему не любят Вербицкого? Хороший математик, преданный струнам.
Аноним 29/12/15 Втр 13:17:22 #338 №322122 
>>322119
Спасибо за попытку, но видимо я не до конца выразил мысль о серьезном образовательном пробеле в области математики, и кроме комплексных чисел схемотехника изобилует другими более злоебучими операциями.
Аноним 29/12/15 Втр 13:30:23 #339 №322123 
>>322051
Neuroscience.

>>322052
Но как? Мне просто хочется врубаться в более сложные концепты, но там неизбежно тонны вышмата во все поля.
Аноним 29/12/15 Втр 13:39:22 #340 №322124 
>>322112
Вообще-то комплексные числа входят в школьную программу. Просто ты, видимо, учился в школе для даунов по урезанным версиям учебников. Скачай любой курс школьных учебников с пометкой "профильный уровень". В основном тебе нужна так называемая алгебра. Профильный школьный курс содержит: наивную теорию множеств, азы теории многочленов и полей (в виде "выражений" и действий с ними), азы теории вероятностей, арифметику комплексных чисел (сложение, умножение и плоскость Аргана), азы дифференциального и интегрального исчисления (в так называемой формулировке де Лопиталя). Если не хватит - Письменный, "Конспект лекций по высшей математике". Если не хватит - Демидович, "Краткий курс высшей математики". Если не хватит - Винберг, "Алгебра", Зорич, "Математический анализ".
Аноним 29/12/15 Втр 13:41:52 #341 №322125 
>>322124
>Просто ты, видимо, учился в школе для даунов по урезанным версиям учебников
А я учился в школе для даунов, но самостоятельно обмазался комплексными числами. Очень нравились все необычные понятия.
Аноним 29/12/15 Втр 13:42:53 #342 №322126 
>>322123
Математика охуенна. Просто русскоязычные книги - плохие, в них символьная абракадабра вместо идей и концепций. Это следствие культурной изоляции. Ты можешь в английский?
Аноним 29/12/15 Втр 14:16:08 #343 №322128 
Напомните, почему программисты считаются говном?
Аноним 29/12/15 Втр 14:21:11 #344 №322129 
Поясните про производные и интегралы. Считать их я могу, благо формулы вывели за меня, но не понимаю сути.
Про производные мне уже пояснили тут, тангенс угла в треугольнике построенном на пределе функции в выбранной точке. Хуево выразил, ну да ладно.

Про интеграл теперь. Интеграл это сумма пределов последовательности функции.
Определенный интеграл это площадь трапеции под криволинейной функцией, ограниченной некоторыми значениями.
Неопределенный интеграл это интеграл в котором производная по верхнему пределу равна подинтегральной функции.
Все правильно написал?
Аноним 29/12/15 Втр 14:42:04 #345 №322130 
>>322129
В математику нет царской дороги. Либо читай книги и изучай доказательства теорем, либо иди нахуй.
Аноним 29/12/15 Втр 14:44:50 #346 №322131 
>>322130
Неты.
Самообучение слишко сложное для меня, не у кого спросить непонятную хуйню.
Аноним 29/12/15 Втр 14:48:46 #347 №322132 
>>322129
через физику легко понять. Производная скорости - ускорение, интеграл - расстояние (всё по времени dt)
Аноним 29/12/15 Втр 15:24:50 #348 №322134 
>>322130
Такие как ты во всем петушиную дрогу выбирают.
Аноним 29/12/15 Втр 15:37:53 #349 №322136 
>>322129
про производную с треугольником пояснял я. теперь про интегралы. значится на самом деле вот эта вот операция взятия производной, это, как ты можешь догадаться, процесс разбиения любой функции на бесконечное чесло бесконечно малых кусочков. по другому еще зовется дифферинциированием. помнишь как строили треугольник? когда подымали с икса пермендикуляры к этой бесконечномалой гипотинузке, мы как бы выделили бесконечно маленькую трапецию. у которой низ это кусок оси иксов, верх та самая гипотинузка, а бока это перпендикуляры. и таких трапеций бесконечно много. бесконечно маленьких. у каждой из них есть какая то площадь. малюсенькая. процесс интегрирования это суммирование площадей этих бесконечно маленьких трапеций. ни больше и ни меньше. типа диференциированием разбили сперва всю большую, неизвестную нам площадь криволинейной трапеции(у которой например одна из сторон не отрезок прямой, а кривая, кусок параболы или еще чего). а потом проинтегрировали - сложили все кусочки, и получили большую площадь. другими словами интегрирование это как ряд суммы, только каждый член этого ряда бесконечно мал. когда он не бесконечно мал пишут знак суммы(та самая с греческая, похожая на Е). а когда бесконечно мал то пишут интеграл. другой сути у процесса интегрирования нет. если дальше будешь учится матану, будешь обьемы считать и прочую хуйню через эти интегралы, а по сути считать сумму бесконечно малых слагаемых, но зато слагаемых этих будет бесконечно много. как это возможно? а вот весь матан и посвящен обоснованию возможности считать именно так.
Аноним 29/12/15 Втр 15:40:33 #350 №322138 
14513928337920.png
Как пятое решить? Расишите все максимально подробно чтобы я понял.
С первого по четвертое ответы
1. 1/3
2. ~0.110
3. 0.5
4. два корня из трех минус два корня из еденицы.
Аноним 29/12/15 Втр 15:44:59 #351 №322140 
>>322138
Взять интеграл от 0 до b. Уровнение прямой y=кх. k = h/b. Как-то так.
Аноним 29/12/15 Втр 15:48:16 #352 №322141 
>>322131
а все-же, где задавать дуратские вопросы?
изучал-изучал, а доказать расходимость:
X_n = (-1)^1 + (-1)^2 + ... + (-1)^n
n -> inf

не могу. или могу, но не понимаю.
могу выделить две подпоследовательности.
при
n = 2k, -> inf.
n = 2k + 1, -> -inf
даже достаточно выделить одну такую последовательность. так-как все подпоследовательности сходящейся последовательности сходятся, то в данном случае это не так, а значит последовательность не сходится.

но не понимаю почему.
(1 - 1) + (1 - 1) + ... = 0
я же могу поставить каждому 2k в соответствие 2k + 1.
и могу выделить подпоследовательность сумм элементов 2k и 2k + 1, которая будет сходиться к нулю.

вот какие книги я использую:
1. Мордкович, Алгебра и начала
2. Ильин и Позняк, Основы
3. Зельдович, Высшая математика для начинающих
а этот пример суммы нашел в книге Abbott, Understanding analysis.
Аноним 29/12/15 Втр 15:49:00 #353 №322142 
>>322129
Извини, но это уже откровенная толстота. Работай тоньше.
Аноним 29/12/15 Втр 15:51:24 #354 №322143 
>>322138
никакого труда там нет. идея в том что тебя попросили проинтегрировать функцию y=kx+b. именно этой функцией, как ты знаешь, описывается уравнение прямой линии. а именно - гипотинузы твоего треугольника. первый катет будет при этом куском оси х. а второй катет будет перпендикуляром, поднятым из точки х=b. только учти что в уравнении y=kx+b конечно же не та b что и в задаче. более того, в твоем случае b в уравнении равно будет 0, потому, что этот б описывает насколько график функции сместится по оси иксов от начала координат, но исследуемая гипотинуза начинается в 0, т.е. сдвига нет, и b=0. попробуй решить, т.е. сперва вывести нужное уравнение, потом взять с него неопределенный интеграл. и увидеть что он равен той самой формуле для площади треугольника. если не получится спрашивай что не получается
Аноним 29/12/15 Втр 15:53:07 #355 №322144 
>>322136
Хммм, ну про то что интеграл это сумма бесконечного количества бесконечно малых я еще в википедии прочитал пока был на первом курсе, когда начались проблемы с ними.
Тобишь, я понимаю что интеграл через предел/бесконечно малые выражается, но я не могу его взять в смысле воображения. Ну разбил я на бесконечное количество кусочков, ну проссуммировал. Не понимаю почему результат должен различаться, и реально различается, между интегрированием и суммированием без разбиения. Я чувствую что ответ где то рядом, но не могу его осознать.
Аноним 29/12/15 Втр 15:56:50 #356 №322145 
>>322141
К чему тут последовательность суммирования минус еденицы в степени? Тут же результат зависит от четности степени.
У меня как раз зельдович, но я не догоняю все равно.
Аноним 29/12/15 Втр 16:12:35 #357 №322147 
>>322112
> Обычные учебники не канают т.к. предполагается наличие препода, который будет объяснять сложные, непонятные моменты.
Нет, они предполагают определенный уровень математической культуры у читателя. То есть, если вместо доказательства написано "очевидно", то после недолгих размышлений тебе должно стать очевидно. Если не становится, то тебе еще рано читать этот учебник.
Советую начать вот с этого
http://www.mccme.ru/free-books/yaschenko/v08book-08.pdf
Аноним 29/12/15 Втр 16:13:16 #358 №322148 
>>322143
Насколько я понял икс уже за нас проинтегрировали.
Получается икс квадрат бэ делить на два, что ни разу не вяжется с формулой площади треугольника, або там квадратов нет.
Аноним 29/12/15 Втр 16:20:10 #359 №322151 
>>322138
Лучше в /un/ спроси, там тебе помогут, а здесь это никому неинтересно.
Аноним 29/12/15 Втр 16:28:30 #360 №322152 
>>322144
>Не понимаю почему результат должен различаться, и реально различается, между интегрированием и суммированием без разбиения

> другими словами интегрирование это как ряд суммы, только каждый член этого ряда бесконечно мал. когда он не бесконечно мал пишут знак суммы(та самая с греческая, похожая на Е). а когда бесконечно мал то пишут интеграл. другой сути у процесса интегрирования нет.
Аноним 29/12/15 Втр 16:32:43 #361 №322154 
>>322148
ты блин не понимал а взял бы да посчитал то, про что я писал. сперва бы вывел формулу y=kx. потом понял что для суммы площади надо взять определенный интеграл, с пределами от 0 до b по иксу. и получил бы формулу kx^2/2 . далее задался бы вопросом как найти ебаную k. почитал бы в интернете. она у тебя не известна. но её можно выразить через h, которую дали в условии. пока выражать будешь у тебя в знаменателе x получится и x^2 превратится в просто x. а k превратится в h. но ведь ты считал определенный интеграл? поэтому x превращается в b. но ты не хочешь считать, ты хочешь говорить что все понял и нихуя не работает. а по сути - не понял и даже не пытался
Аноним 29/12/15 Втр 16:44:13 #362 №322159 
>>322154
Я и не говорил что я все понял.Я наоборот ничего не понял.
Ну вот прямую я проинтегрировал, получил ка икс квадрат делить на два. Как мне её через высоту выразить? Уравнение прямой подкорректровать чтобы через концы аш и бэ проходила? Тангенс вычислять что ли?
Аноним 29/12/15 Втр 17:08:57 #363 №322166 
>>322159
ну надо начать с вопроса что вообще за говно такое k. а для этого в бить в гугол "уравнение прямой". там дадут ясно понять что Уравнение прямой с угловым коэффициентом

Общее уравнение прямой при B≠0 можно привести к виду

y = k x + b

где k - угловой коэффициент равный тангенсу угла, образованного данной прямой и положительным направлением оси ОХ. http://ru.onlinemschool.com/math/library/analytic_geometry/line/ . далее подумаем как можно выразить этот ебаный тангенс из двух известных. а известен нам один катет - b, и высота - h. ну например давай рассмотрим треугольник, у которого один из катетов будет h, второй катет это кусок гипотинузы. а гипотинуза этого нового треугольника - сторона b. из этого треугольника можем найти тангенс угла. выразив неизвестный катет через изветсную гипотинузу b и известный катет h. по теореме пифагора найдем его, будет он у нас для красоты z. z^2=b^2-h^2. теперь тангенс получается h/z, теперь можно дрочить выражение, зависящее только от h и b чтобы получить искомую 1/2 hb. но возможно мое выражение тангенса не каноничное, какое правильно брать вопрос геометрии, а всех этих азов я конечно уже не помню. давно это было хе хе хе, уж потрудись сам хоть чуть чуть над решением, а то толку будет 0
Аноним 29/12/15 Втр 17:50:43 #364 №322168 
14514006434450.png
Как узнать к чему этот ряд сходиться?

sum [//math:(-1)^n*n^(-1)//] from [//math:1//] to [//math:infinity//]

Аноним 29/12/15 Втр 18:07:00 #365 №322173 
>>322168
Открыть учебник розенталя, тупой мудак.
Аноним 29/12/15 Втр 18:07:47 #366 №322174 
14514016670610.png
>>322168
Ни за что не догадаешься
Аноним 29/12/15 Втр 18:09:54 #367 №322175 
>>322174
А решение?

>>322173
Кстати реквестирую небольшой курс чтобы за вечер мог научиться простые задачки с рядами решать.
Аноним 29/12/15 Втр 18:35:53 #368 №322179 
14514033534260.png
Кстати поясните ещё за это. Что за тема?
Аноним 29/12/15 Втр 18:38:12 #369 №322180 
>>322179
-> >>322130
Аноним 29/12/15 Втр 18:54:08 #370 №322182 
Ну всё, началась сессия.
Аноним 29/12/15 Втр 18:59:44 #371 №322183 
>>322182
кончилась
Аноним 29/12/15 Втр 19:15:39 #372 №322190 
>>322118
Бамп. Почему какому-то жирному дауну отвечают, а меня игнорируют?
Аноним 29/12/15 Втр 19:17:52 #373 №322191 
>>322166
Ясно.
Надо с самого начала перечитывать книжку.
Аноним 29/12/15 Втр 19:20:45 #374 №322194 
>>322118
>>322190
https://openlibrary.org/search?q=Francesco+Severi
Аноним 29/12/15 Втр 19:25:34 #375 №322197 
>>322194
>за далары
)
Аноним 29/12/15 Втр 19:32:57 #376 №322199 
>>322118
Алсо, встретил в этой статье термин "линейная серия". Где можно найти его определение и примеры? Совсем ничего не гуглится.
http://гоппа.рф/Publications/ag_codes.pdf
Аноним 29/12/15 Втр 20:02:37 #377 №322200 
Где можно скачать вот это говно? Понравилось обилие примеров.

https://books.google.ru/books?id=-owiBQAAQBAJ&pg=PA2&lpg=PA2&dq=%D0%B2%D0%B5%D1%87%D1%82%D0%BE%D0%BC%D0%BE%D0%B2+%D0%BE%D1%81%D0%BD%D0%BE%D0%B2%D0%BD%D1%8B%D0%B5+%D0%BC%D0%B0%D1%82%D0%B5%D0%BC%D0%B0%D1%82%D0%B8%D1%87%D0%B5%D1%81%D0%BA%D0%B8%D0%B5+%D1%81%D1%82%D1%80%D1%83%D0%BA%D1%82%D1%83%D1%80%D1%8B&source=bl&ots=MBYxZlRxV2&sig=WG2Z2QweHHkJpTmJudzkgVcrQPc&hl=ru&sa=X&ved=0ahUKEwiPpeODxYHKAhVIlXIKHUZZDiAQ6AEIIjAB#v=onepage&q=%D0%B2%D0%B5%D1%87%D1%82%D0%BE%D0%BC%D0%BE%D0%B2%20%D0%BE%D1%81%D0%BD%D0%BE%D0%B2%D0%BD%D1%8B%D0%B5%20%D0%BC%D0%B0%D1%82%D0%B5%D0%BC%D0%B0%D1%82%D0%B8%D1%87%D0%B5%D1%81%D0%BA%D0%B8%D0%B5%20%D1%81%D1%82%D1%80%D1%83%D0%BA%D1%82%D1%83%D1%80%D1%8B&f=false
Аноним 29/12/15 Втр 20:27:04 #378 №322205 
>>322200
я могу очень постараться и написать скрипт, который сольёт эту книжку из гугла, а потом склеить результат в какое-нибудь джвю. Оно того стоит? Я стану легендой?
Аноним 29/12/15 Втр 20:30:54 #379 №322206 
>>322205
хер там был, гугл починили обход ограничения на просмотр
Аноним 29/12/15 Втр 20:59:32 #380 №322210 
>>322205
Там не все страницы - книга с пробелами. А так для этого уже давно существует Google book downloader скрипт для greasemonkey.
Аноним 29/12/15 Втр 22:05:33 #381 №322224 
>>322118
На первой странице по запросу в Гугле http://bkmd.narod.ru/doc/Lezioni-Severi-ru.pdf
>>322199
То же самое, что и линейная система, как я понимаю.
http://math.stackexchange.com/questions/579732/linear-series-vs-linear-system-on-algebraic-curves
Аноним 29/12/15 Втр 23:26:28 #382 №322232 
>>322122
я посоветовал литру по единственному топику в котором могу советовать
если книжки слишком сложно написаны могу объяснить или посоветовать что другое
компл. числа это далеко не пиздец, там нет ничего по-настоящему сложного
например, в америке их проходят школьники, и никто не умирает. да и тут в физматах про них уже знают, иногда класса с 8
если относится к какой-то теме как к дико сложному колдунству то так оно наверное и будет, - там где все эту элементарную поебень уже давно освоили будешь двигаться со скоростью черепахи или вообще стоять на месте суеверно тараща глаза на "i^2=-1"
Аноним 30/12/15 Срд 11:13:47 #383 №322277 
>>322232
Ну вот как мне представить число и квадрат равно минус еденице?
Аноним 30/12/15 Срд 11:47:52 #384 №322282 
>>322277
а чего там представлять? как ты себе представляешь число корень из двух? вот так же представь что есть некое чесло, которое если возведешь в квадрат то будет 1. но на самом деле эти ваши комплексные числа можно представлять и по другому. вот есть точка на плоскости. у неё есть координаты. одна по игреку, другая по иксу. вот то что по иксу это действительная часть чесла, а что по игреку - мнимая. типа i тут будет как бэ означать принадлежность координаты к игреку. очень хорошо это понимаешь после знакомства с аналитической геометрией, где используются единичные векторы, орты. i можно рассматривать как такую вот орту.
Аноним 30/12/15 Срд 12:09:31 #385 №322284 
Я Бурбаки.
Аноним 30/12/15 Срд 13:57:47 #386 №322300 
>>322277
Любой математический объект - это некоторое множество. Число 5 - множество. Число 17/3 - множество. Число i - множество. Если ты знаешь, что такое множество, то проблем быть не должно.
Аноним 30/12/15 Срд 14:37:06 #387 №322303 
>>322277
>>322282
>>322300
А и не нужно ничего представлять. Прямую вязь с реальностью вообще имеет только и только множество N.
Аноним 30/12/15 Срд 14:40:09 #388 №322304 
>>322126
Да, могу, кидай списокъ.
Аноним 30/12/15 Срд 14:57:13 #389 №322308 
>>322303
Пеано давно последний раз снился?
Аноним 30/12/15 Срд 15:02:13 #390 №322309 
>>322282
Мне на первом курсе говорили про координатную ось. Но я все равно не понял как себе представить и.
Корень из одного, двух, трех.....я могу себе представить. Тут суть не в определении числа, а в знаке.

>>322300
И что такое множество?
Аноним 30/12/15 Срд 17:35:31 #391 №322328 
>>322309
МНОЖЕСТВО ЭТО КОГДА СОКРАЩЕНИЕ ТРАНСЦЕНДЕНТНЫХ ОБЪЕКТОВ АБЕЛЕВОЙ ГРУППЫ ПРОИСХОДИТ ЗА СЧЁТ БЕЗУСЛОВНОЙ ДИСФУНКЦИИ МНИМОЙ КАНОНИЧНОЙ НЁТЕРОВОЙ ФУНКЦИИ СО СТРОГО ПРИВЕДЁННЫМИ УРАВНЕНИЯМИ, СВЯЗАННЫМИ КОРНЯМИ НАД ПОЛЕМ ТОЛЬКО И ТОЛЬКО КОМПЛЕКСНЫХ ЧИСЕЛ КОГЕРЕНТНОГО КВАДРАТИЧНОГО СОСТОЯНИЯ, ПУТЁМ ДИФФЕРЕНЦИРОВАНИЯ СЕПАРАБЕЛЬНЫХ РАСШИРЕНИЙ И ПОЛОЖИТЕЛЬНОГО ФУНКЦИОНАЛЬНОГО ПЕРЕГИБА В ТОЧКЕ СОКРАЩЕНИЯ АНТИСИММЕТРИЧНЫХ МНОГОЧЛЕНОВ С ТОЧНОСТЬЮ ОТ ПОДКОЛЬЦА КОЛЬЦА Б И ОДНОРОДНОЙ ПЕРЕМЕННОЙ В ДРОБНОЙ НЕЧЁТНОЙ ОТРИЦАТЕЛЬНОЙ СТЕПЕНИ В ОБЛАСТИ НЕЗАВИСИМОСТИ АВТОРМОРФИЗОВ НЕСТРОГО ОТ ОБЛАСТИ ОПРЕДЕЛЕНИЯ КОМПОЗИТА РАСШИРЕНИЯ ГАЛУА.
Аноним 30/12/15 Срд 17:43:37 #392 №322329 
>>322309
Технически - это строка символов некоторой аксиоматической формальной системы.
Более содержательно - умозрительный объект из того сорта объектов, которые подчиняются аксиомам Цермело-Френкеля или альтернативным.
Аноним 30/12/15 Срд 17:43:44 #393 №322330 
14514866245250.png
>>322328
Почему аксиомы ZFC не определяют множество?
картинка с петухом у параши, которому снится Кантор, Цермело и Френкель.png
Аноним 30/12/15 Срд 17:57:44 #394 №322332 
>>322330
С чего ты взял, что не определяют?
Аноним 30/12/15 Срд 17:58:53 #395 №322333 
>>322332
это такой сцаевский юмор.
Аноним 30/12/15 Срд 18:04:54 #396 №322334 
>>322141
Ты не имеешь права переставлять члены ряда, если он расходится.
Аноним 30/12/15 Срд 18:13:35 #397 №322336 
ПОЧЕМУ ВЕРБИЦКИЙ ЁБАННЫЙ? ПОЧЕМУ ВЫ НЕ ХОТИТЕ РАБОТАТЬ СО СТРУНАМИ?
Аноним 30/12/15 Срд 18:29:44 #398 №322339 
Пацоны, а как додумались до гомологий топологических пространств? Определение же совершенно какое-то комбинаторное и вообще совсем неочевидно, почему это будет инвариантное что-то.
Аноним 30/12/15 Срд 18:30:42 #399 №322340 
>>322336
Потому что трахал себя резиновым членом и описывал ощущения.
Аноним 30/12/15 Срд 18:46:32 #400 №322344 
Пацаны, расскажите, где еще применяется теория Галуа кроме как в криптографии и теории кодирования. Очень много внимания ей уделяется, даже какие-то топологические обобщения придумывают.
Аноним 30/12/15 Срд 18:55:26 #401 №322345 
>>322344
Накрытия Галуа.
Аноним 30/12/15 Срд 20:52:55 #402 №322379 
Матаны, где можно почитать про историю развития математики? Тобишь, как открыли бесконечно малые, комплексные числа, мамку опа, поля, топологии с гомотопиями, т.д.
Аноним 30/12/15 Срд 20:53:41 #403 №322380 
14514980211170.png
Из чего состоит алгебраическое замыкание конечного поля? Как построить его на бумажке? Как представить себе многочлен с коэффициентами из декартова произведения алгебраического замыкания конечного поля?
Аноним 30/12/15 Срд 20:56:26 #404 №322381 
>>322380
Замкнул тебя на ноль, даун.
Читай фихтенгольца, там же все очевидно.
Аноним 30/12/15 Срд 20:58:53 #405 №322383 
>>322381
Иди нахуй.
Аноним 30/12/15 Срд 21:36:50 #406 №322390 
14515006106880.png
Так, я бегиннер и нихуя не вижу разницы между if и only if.
СУКА ТРЕТИЙ ДЕНЬ УЖЕ ЕБУСЬ С ЭТОЙ ПАРАШЕЙ
Аноним 30/12/15 Срд 22:01:17 #407 №322398 
>>322390
1. Если щёлкнуть кобылу в нос, то она махнёт хвостом.
Кобыла махнула хвостом. Значит ли это, что её щелкнули в нос? Вообще говоря нет, потому что кобыла могла махнуть хвостом по многим другим причинам.

2. Кобыла махнёт хвостом, только если щёлкнуть её в нос.
Кобыла махнула хвостом. Значит ли это, что её щелкнули в нос? Да, значит.
Аноним 30/12/15 Срд 22:03:03 #408 №322400 
>>322383
Сходил тебе нахуй в кадык с левой и в висок КАРОНЫМ-ПАХАРОНЫМ с правой, ничтожество.
Аноним 30/12/15 Срд 22:17:24 #409 №322402 
>>322400
Просто ссу тебе в волосы, графоман, кусок говна.
Аноним 30/12/15 Срд 22:38:41 #410 №322406 
>>322398
Спасибо, стало понятнее.
Аноним 30/12/15 Срд 22:58:15 #411 №322408 
Читаю фихтенгольца, а он мне начинает за поля втирать и я теряю суть разговора. Это норма?
Аноним 30/12/15 Срд 23:02:46 #412 №322409 
>>322408
Двачую, пидоры какие-то. Наверно, на полях работают, колхоз ебаный, хуле они в математику лезут со своими полями?
Аноним 30/12/15 Срд 23:06:22 #413 №322410 
>>322409
Ну за поля я загнул, но он с первых страниц начинает про сечения говорить, множества, пустоту-непустоту не дав определения что это такое за хуйня.
Аноним 30/12/15 Срд 23:23:24 #414 №322411 
>>322410
Аноны, порекомендуйте что-нибудь кошерное из Фихтенгольца. Программисты нахваливают какие-то разделы из его книг, мол, там прямо самый смак. А я анализом не сильно интересовался, я больше алгебру и геометрию люблю.
Аноним 30/12/15 Срд 23:26:51 #415 №322412 
Аноны, объясните, пожалуйста, человеческим языком что такое декартово произведение множеств.
Аноним 30/12/15 Срд 23:39:22 #416 №322414 
>>322412
Если X, Y, Z - множества, то декартово произведение X x Y x Z состоит из векторов (x, y, z), координаты которого взяты только из соответствующего множества.
Аноним 31/12/15 Чтв 00:38:05 #417 №322419 
О, чуете как быстро по треду распространяется запашок сдавшего 40 интегральчиков на предновогодней части зимней сессии воняющего спиртягой и спермой первокурсника мухосранского ПриМата?
sageАноним 31/12/15 Чтв 00:44:47 #418 №322420 
>>322412
это легче всего представить как таблицу. Смотри, для тебя даже картинку на википедии сделали
https://upload.wikimedia.org/wikipedia/commons/thumb/4/4e/Cartesian_Product_qtl1.svg/789px-Cartesian_Product_qtl1.svg.png

Декартово произведение множества А на множество B
это просто множество всех возможных пар (a,b)
где а принадлежит А, b принадлежит B
и элемент из А стоит первым
Аноним 31/12/15 Чтв 00:56:24 #419 №322423 
>>322419
Пруфани свой диплом НМУ, денегерат.
sageАноним 31/12/15 Чтв 01:06:21 #420 №322425 
>>322420
т.е. элементы декартового произведения 2 множеств - это пары
примеры декартовых произведений:
произведение двух отрезков это прямоугольник
произведение двух обычных числовых прямых это плоскость
произведение окружности на отрезок это цилиндр
произведение окружности на окружность это тор (фигура кот. выглядит как бублик или пончик)
http://math.siomax.ru/Sets
вот тут вроде норм написано с главы 3.3, хотя я не читал
>>322277
Расслабься. Для начала посмотри вот мультик, там про это есть в одной из серий. Я посмотрел почти до конца в свое время, не жалею.
www.dimensions-math.org/Dim_regarder_RU.htm
Потом попробуй книжку. Советую начать с "Теорема абеля в задачах и решениях". Она не совсем по теме, но комплексные числа там будут, и вообще пойдет на пользу даже если не закончишь. Просто привыкнешь к немного более общей математике чем то говно что пихают в школе.
Те две книги что я рекомендовал выше тоже сойдут, хотя Понарина я только листал в нужных моментах, мб читать его подряд и не стоит. Но вот гиперкомплексные числа вполне ок. Если слишком сложно - ну что ж, тогда не обижайся но придется читать что-то для детей. Например Гельфанд-Шень алгебра.
Скоро вся эта восторженность пройдет, ты будешь работать с комплексными числами и удивляться когда говорят что это что-то сложное.
i это просто символ, про него нужно знать что он в квадрате дает -1
Аноним 31/12/15 Чтв 01:11:22 #421 №322427 
Мы тут любим показывать свою циничность, бравируем смехом над трупами и изнасилованными детьми — но реальность в том, что нам похуй. мы ощущаем себя такими лузерами, что всякое говно вокруг безмерно чище и выше нашего самомнения. Нам не нужны люди, людское тепло… Нет, оно нам надо, это заветная тайная мечта, в которой ни один битард не признается даже на анонимной имиджборде через тор и прокси (у меня такой мечты, например, нет). Мечта о понимании, о том, чтобы нам делали хорошо, любили, нежили. Чтобы от нас ничего не хотели, и мы были бы всем нужны… Оставьте нас в покое! Но не оставляйте нас… нам плохо. Мы не гордые «санитары интернета». Мы душевнобольные, хроничести и тяжко. У нас не бывает приступов — лишь ноющая боль души, которую мы привычно глушим лулзами и цинизмом. Цинизм — это всегда крик «спасите меня» — и ядовитые шипы навстречу тем, кто попробует шагнуть ближе… Новый год — момент катарсиса. Момент, когда каждый битард озлобляется, становится не по-битардовски истеричен. Новый год — это когда каждый из нас понимает, какая это глупость — ёлка, новый год, смех и шутки, друзья, семья, что всё это хуйня по сравнению с гомонеграми; да и гомонегры тоже хуйня, мы одни и нас никто не любит. Потому что те, кто нас может любить (и быть может пытается показать симпатию) вызывает в нас иррациональный страх. А если это семья — то раздражение и ненависть. Мы плохие (неудачники, уроды, девственники, задроты) — и потому мы бежим от тех, кто проявляет к нам чувства, потому что мы лишь острее ощущаем свою мерзостность. Потому мы гордимся одиночеством и задротством. Но на самом деле мы плачем. Где-то там, внутри, сжавшись в комочек, плачем о нашей жизни. Которая уже проёбана, и день ото дня всё плотнее забивает канализацию, куда мы её спустили… когда? Наверное, когда в школе не понимали как говорить со сверстниками. Или когда дрочили, так и не подойдя к девочке (мальчику), а о том шансе потом вспоминали триста раз, издрочили в фантазии этот момент до выцветшей пустышки, и так и не сделали шаг. Потому что ходить не умеем. Не умеем ходить к людям. Ходить с людьми.

Что такое битарды? Это фактически отбросы общества. Люди, которые не смогли найти в нем места и которые всей душой (хотя подчас этого и не сознавая) его ненавидят. А Два.ч это берлога битардов. Зачем все нужно? Just for lulz. Просто так зачмырить кого-то до самоубийства. Просто так сломать кому-то жизнь. Просто посмеяться. «Потому что мы ненавидим всех других. Потому что мы другие. Мы не из этого остального быдла с ТП, планктоном и быдлом. И поэтому мы можем резать, жечь и убивать, издеваться над моралью и ценностями, смеясь ломать навсегда людям жизни. Это Два.ч. Мы вас ненавидим, но мы просто такие. Любите нас, пожалуйста.
Аноним 31/12/15 Чтв 03:36:29 #422 №322439 
>>322414
>>322420
>>322425
Спасибо, анончики!
Аноним 31/12/15 Чтв 04:07:24 #423 №322441 
14515240443650.jpg
>>322427
ХУЛИ ОНИ СОБРАЛИСЬ ВМЕСТЕ И ТУПЯТ КОЛЛЕКТИВНА?
ИМ ОБЯСНЯЕШ ЧТО НИ ДЕБИЛЫ, ОНИ НЕ ПОНИМАЮТ
МАКСИМУМ НА ЧТО МОЖНО РАСЧИТЫВАТЬ В ТАКОМ КОЛЛЕКТИВЕ- ДОЛЖНОСТЬ МАЛЬЧИКА ДЛЯ УНИЖЕНИЙ
ПОКА ОНИ ВСЕ НЕ ПЕРЕДОНУТ С ГОЛОДУ

Аноним 31/12/15 Чтв 12:25:43 #424 №322484 
>>322423
Нет, ты протрезвей сначала, антоша, и одежду смени - проссанная вся.
Аноним 31/12/15 Чтв 13:48:51 #425 №322495 
>>322380
Бамп. Нахуя нужны алгебраические замыкания, если их даже нельзя ни представить, ни построить на бумажке, и остается в них только веровать? Я люблю что-нибудь вычислять, мне нравятся сложно устроенные объекты, и алгебраическое замыкание - как раз такой интересный объект. Почему я не могу им обмазаться?
Аноним 31/12/15 Чтв 14:05:20 #426 №322501 
Поясните за фихтенгольцовские поля, множества, хуежества, а то я дальше первых двадцати страниц не могу читать.
Аноним 31/12/15 Чтв 14:32:53 #427 №322505 
Как развить математическую культуру тм?
Пытался ебать себя дилдой в жопу как Вербитский, но что-то не чувствую культурного прозрения зато в области сфинктера приятно.
Аноним 31/12/15 Чтв 16:33:06 #428 №322523 
>>322495
http://planetmath.org/algebraicclosureofafinitefield
Аноним 31/12/15 Чтв 16:36:38 #429 №322525 
>>322380
В программировании есть концепция инкапсулирования. Ты знаком с ней? Мы забываем внутреннюю структуру объекта и рассматриваем объект как чёрный ящик, с которым можно совершать лишь манипуляции из точно перечисленного набора. Нет ничего странного в том, чтобы соорудить из многочленов некоторое поле, а потом забыть о том, что элементами этого поля являются вообще-то многочлены и начать думать об элементах просто как о скалярах без внутренней структуры. Многочлен, коэффициенты которого взяты из некоторого поля многочленов, - вполне разумная и обозримая штука.
Аноним 31/12/15 Чтв 16:41:07 #430 №322526 
>>322525
Чтобы инкапсулировать данные в объекте, их сначала надо построить. В математике аналогично.
Забывать стоит, когда ты себе хорошо представляешь объект, но для удобства лучше не думать о его устройстве. Также мне непонятно, для чего нужны многочлены задающие кривые, коэффициенты которых являются многочленами. В криптографии многочлены имеют только числовые коэффициенты. Тогда зачем вообще морочить голову сложными абстракциями, которые нельзя себе представить? Какая в этом необходимость? Про необходимость никто не говорит, поэтому у любителей конструктивного похода возникают брейнфаки при попытке залезть во внутренности объекта.
Аноним 31/12/15 Чтв 16:43:37 #431 №322527 
>>322523
Во, это поинтереснее того, что я читал.
Аноним 31/12/15 Чтв 17:11:54 #432 №322529 
>>322390
Че непонятного? Построим таблицу истинности A => B:
A, B, A => B
0, 0, 1
0, 1, 1
1, 0, 0
1, 1, 1

Из таблицы видно, что A может быть равно 1 только если B = 1. То есть "A только если B".
Аноним 31/12/15 Чтв 17:39:57 #433 №322531 
>>322526
Предположим, мы желаем доказать существования поля B, являющегося расширением поля A и обладающего некоторыми свойствами. Есть типичный способ. Мы строим из поля A кольцо многочленов, вкладываем его в поле частных, забываем о внутренней структуре и получаем поле A'. Его элементы - сложные объекты, но мы забываем их структуру и думаем просто как о скалярах. С полем A' делаем аналогичные манипуляции и получаем поле A''. Потом поле A''', A'''' и так далее, и в итоге получаем поле B.
Аноним 31/12/15 Чтв 19:09:40 #434 №322541 
>>322525
>Многочлен, коэффициенты которого взяты из некоторого поля многочленов, - вполне разумная и обозримая штука.
И, то есть, мы можешь раскрыть скобки, привести подобные члены и получить привычный нам многочлен, у которого коэффициенты - числа?

Например, как-то так: (ax+b)xy + (cx + d)yz = axxy + bxy + cxyz + dyz
Аноним 31/12/15 Чтв 20:30:50 #435 №322549 
>>322541
Нет.
Аноним 31/12/15 Чтв 21:05:03 #436 №322553 
>>322549
А подробнее можно?
Аноним 31/12/15 Чтв 21:28:47 #437 №322559 
>>322553
Что именно тебя интересует? Ты же не смотришь на число 17 как на последовательность рациональных чисел (хотя должен, это стандартная конструкция поля вещественных чисел), ты думаешь о нём как о скаляре. Ну вот и на многочлены смотри не как на многочлены, а как на не имеющие внутренней структуры числа новой числовой системы, расширяющей имеющуюся.
Аноним 31/12/15 Чтв 22:46:23 #438 №322569 
14515911838680.png
Какие ещё есть варианты? Вольфрам альфа уже 15 минту думает. А мне бы просто таблицу значений до 1000 хотябы.
Аноним 31/12/15 Чтв 22:58:48 #439 №322571 
>>322569
Напиши прогу.
Аноним 31/12/15 Чтв 23:03:04 #440 №322573 
,kznm
Аноним 31/12/15 Чтв 23:23:47 #441 №322574 
>>322571
Я?
Аноним 01/01/16 Птн 00:06:54 #442 №322576 
Хуле все молчат? Лежите мордой в салате, картофаны?
Аноним 01/01/16 Птн 00:11:05 #443 №322577 
>>322576
Аутист штоле? А ну пошел быстро накатил, вмсето того, чтобы тут сидеть.
Аноним 01/01/16 Птн 00:23:06 #444 №322578 
Анончики, я просто хочу поздравить вас с Новым Годом. И сказать спасибо, что вы помогали мне своими советами и разъяснениями в 2015. Без вас я бы никогда не осилил общую алгебру.
Аноним 01/01/16 Птн 00:30:52 #445 №322580 
>>322576
Меня только что оттрахал мой кун, например.

>>322578
И тебя с ним. Пожалуйста.
Аноним 01/01/16 Птн 00:34:24 #446 №322581 
>>322577
А я про функциональные поля читаю.
>>322580
>Меня только что оттрахал мой кун, например.
Надеюсь, ты гей, а не тян?
Аноним 01/01/16 Птн 00:36:32 #447 №322582 
>>322581
Гей.
Аноним 01/01/16 Птн 00:37:37 #448 №322583 
Кстати, я только что умозрительно подрочил на няшку. Это гораздо приятнее, чем просто гонять шкурку. Когда с закрытями глазами придумываешь эмоции твоей любимой 2д тян, проецируешь на себя, пытаешься почувствовать то, что чувствует она, сам изнемогаешь в это время от высшей степени близости и возбуждения и наконец довершаешь дело божественным фапом.
Аноним 01/01/16 Птн 10:56:36 #449 №322592 
>>322582
Володин?
Аноним 01/01/16 Птн 10:58:30 #450 №322594 
Если взять многочлен с алгебраическими коэффициентами, может ли его корнем быть любое действительное число?
Аноним 01/01/16 Птн 11:27:12 #451 №322595 
>>322594
У многочлена степени n не более n корней, уже поэтому нет.
Аноним 01/01/16 Птн 11:47:11 #452 №322596 
>>322595
ровно n.
Аноним 01/01/16 Птн 11:50:16 #453 №322597 
>>322596
Возможны кратные.
Аноним 01/01/16 Птн 11:53:02 #454 №322598 
>>322597
Это не отменяет их количества.
Аноним 01/01/16 Птн 11:54:00 #455 №322599 
>>322598
У многочлена степени n не более n корней. Это истинно.
Аноним 01/01/16 Птн 12:02:43 #456 №322600 
>>322599
>не более n корней
не более n _различных_ корней
Аноним 01/01/16 Птн 12:03:54 #457 №322601 
>>322600
Это тоже истинно. Хотя из равенства следует "не более", поэтому зачем уточнять.
Аноним 01/01/16 Птн 12:38:57 #458 №322602 
14516411375370.webm
>>320047 (OP)
Аноним 01/01/16 Птн 13:16:35 #459 №322605 
>>322602
ЗАРЕКОМЕНДУЙ СБОРКУ ДЛЯ РАСЧЕТА ПРОТЕИНОВ!
Аноним 01/01/16 Птн 16:37:23 #460 №322610 
14516554433030.jpg
Помоги, пожалуйста, мозг просто отключается.

Есть 160 шаров, из которых 40 красные, а остальные белые. Найти вероятность p(m), что среди наугад выбранных m (m <= 160) шаров хотябы один окажется красным.
Аноним 01/01/16 Птн 17:31:18 #461 №322624 
>>322569
Тебя асимптотика интересует или что?
Аноним 01/01/16 Птн 18:16:15 #462 №322639 
>>322610
Тут, по всей видимости, выбор без возвращения.
N=160, k=40
имеет смысл рассматривать только случай, когда m <= N-k, поскольку иначе p(m)=1
Проще искать вероятность того, что среди m выбранных наугад шаров нет красных
По формуле условной вероятности,
1-p(m) = ( (N-k)/k ) ( (N-k-1)/(k-1) ) ... ( (N-k-(m-1))/(k-(m-1)) ) = (N-k)! (N-m)! / (N-k-m)! N! = C_{N-k}^m / C_N^m
Аноним 01/01/16 Птн 18:41:04 #463 №322651 
>>322595
Это справедливо к рациональным коэффициентам, а я сказал алгебраическим.
Аноним 01/01/16 Птн 19:09:01 #464 №322661 
>>322651
Это справедливо с коэффициентами из любого поля: если у многочлена $x^n+a_nx^{n-1}+...+a_1x^0$ есть различные корни $b_1,...,b_n$, то он обязан быть равен $(x-b_1)(x-b_2)...(x-b_n)$, что несложно показать индукцией по $n$ c использованием теоремы Безу для обоснования перехода.
Аноним 01/01/16 Птн 19:17:05 #465 №322662 
>>322610
Очевидно, что
1 - (C из 120 по n) / (C из 160 по n)
Аноним 01/01/16 Птн 21:48:45 #466 №322674 
Есть сфера диаметром 2 метра. От этой сферы отрезали сектор, центр окружности которого проходит на расстоянии 1/5 диаметра от поверхности сферы. Как найти диаметр окружности по которой резали?
Аноним 01/01/16 Птн 22:03:57 #467 №322676 
>>320047 (OP)
Вопрос по парадоксу Рассела.
Вот например формулировка из вики: "Пусть K — множество всех множеств, которые не содержат себя в качестве своего элемента. Содержит ли K само себя в качестве элемента? Если предположить, что содержит, то мы получаем противоречие с "Не содержат себя в качестве своего элемента". Если предположить, что K не содержит себя как элемент, то вновь возникает противоречие, ведь K — множество всех множеств, которые не содержат себя в качестве своего элемента, а значит должно содержать все возможные элементы, включая и себя."
>Пусть K — множество всех множеств, которые не содержат себя в качестве своего элемента
Зачем это требование? Разве при данном определении K и есть то самое искомое множество всех множеств? А если найдется множество, содержащее себя в качестве элемента, ведь невозможность привести конкретный пример не доказывает его отсутствие.
Проучился пол года на математическом факультете, но до сих пор не могу въехать в логику доказательства невозможности существования множества всех множеств, помогите плз.
Аноним 01/01/16 Птн 22:25:20 #468 №322678 
>>322676
Твоя ошибка в том, что ты думаешь, что парадокс Расселла доказывает несуществование множества всех множеств. На самом деле парадокс Расселла доказывает, что не всякое свойство является коллективизирующим, точнее, свойство "не содержать себя в качестве своего элемента" не является коллективизирующим.

Напомню, что свойство (предикат) P(x) является коллективизирующим, если совокупность всех таких x, для которых P(x) истинно, является множеством.

Идея, что парадокс Расселла убивает конкретно множество всех множеств, встречается только в быдлолитературе.

Несуществование множества всех множеств M называется антиномия Кантора. А именно. По известной теореме Кантора, мощность множества всех подмножеств P(M) множества M должна быть строго больше мощности M. Но M - множество всех множеств, поэтому P(M) - подмножество M и потому мощность P(M) не превосходит мощность M. Проблема.
Аноним 01/01/16 Птн 22:49:35 #469 №322682 
>>322678
Большое спасибо.
Вот формулировка парадокса в конспекте лектора: "Парадокс Рассела: множество всех объектов невозможно". Получается, множество всех множеств, которые не содержат себя в качестве своего элемента это лишь частный случай множества всех объектов. Мы приходим к противоречию, что такого множества быть не может. Таким образом, мы привели пример, когда множество всех объектов невозможно. НО ЭТО ТОЛЬКО В ТОМ СЛУЧАЕ, КОГДА ОБЪЕКТАМИ МЫ СЧИТАЕМ МНОЖЕСТВА, НЕ СОДЕРЖАЩИЕ СЕБЯ В КАЧЕСТВЕ ЭЛЕМЕНТА. Где же здесь переход от конкретного к общему случаю?
Аноним 01/01/16 Птн 23:30:05 #470 №322695 
14516802057900.png
>>320086
Бля. Уже хотел было выебнуться, написав, как это изи, но обосрался, потому что не могу даже действия сделать без комплексных чисел.
Я хуй проссу, как на >>320112 получается (t^2 - t + 1)(t^2 + t + 1).
Аноним 01/01/16 Птн 23:30:52 #471 №322696 
..определение натуральных..
Аноним 01/01/16 Птн 23:44:49 #472 №322702 
>>322695
Извините, но это же и правда легко, хоть и долго. Выделяем полный квадрат и раскладываем как разность квадратов.
Аноним 01/01/16 Птн 23:53:22 #473 №322710 
>>322696
...петух у параши...
Аноним 02/01/16 Суб 00:18:31 #474 №322714 
Гайз, поясните, плиз, разложение многочленов на множители и упрощение выражений - херня для школьников, или в высшей математике это тоже важно? Профессиональные математики как к этому относятся?
Аноним 02/01/16 Суб 01:35:37 #475 №322727 
>>322714
они только этим и занимаются - упрощают выражения
Аноним 02/01/16 Суб 04:13:46 #476 №322749 
>>322727
А как же общая алгебра? Там вообще ведь есть только абстрактные сущности типа групп и категорий, что там упрощать?
Аноним 02/01/16 Суб 06:08:39 #477 №322751 
>>322749
ВЫРАЖЕНИЯ
Аноним 02/01/16 Суб 06:27:37 #478 №322752 
>>322751
Приведи пример, пожалуйста.
Аноним 02/01/16 Суб 10:52:36 #479 №322769 
>>322674
Сформулируй точнее, не понятно же.
Аноним 02/01/16 Суб 11:06:15 #480 №322770 
>>322769
>непонятно
фикс
Аноним 02/01/16 Суб 11:09:28 #481 №322772 
14517221688850.gif
>>322674
ИЗИ
Будем пользоваться радиусом сферы: R = 1м
Отмеряем расстояние от центра окружности до центра сферы:
R - (1/5)*d = 1 - 0.4 = 0.6м
Вопщим найденное расстояние, радиус сферы и радиус окружности образуют прямоугольный треугольник, в котором радиус сферы - гипотенуза.
Считаем по теореме Пифагора радиус окружности:
sqrt(1^2 - 0.6^2) = 0.8м
Её диаметр тогда равен 1.6м.
Вроде ничего не напутал. По крайней мере, ты теперь знаешь, каким образом считать такое дерьмо.
Но нахуя написано про сектор - мне не понять.
Аноним 02/01/16 Суб 11:11:03 #482 №322773 
>>322772
Да, ещё и нигде не указано, что окружность - это сечение сферы. Если она не является таковым, то хуй знает, как решать.
Аноним 02/01/16 Суб 11:57:08 #483 №322776 
>>322714
Это важно, но решать задачи в духе "упростите выражение" точно не стоит. Нужно доказывать теоремы и выводить формулы, а техника со временем придет сама.
Аноним 02/01/16 Суб 12:07:48 #484 №322781 
>>322714
Если ты школьник, советую начать с этого
http://www.mccme.ru/free-books/yaschenko/v08book-08.pdf
Аноним 02/01/16 Суб 13:11:51 #485 №322796 
Обьясните как для дауна доказательство леммы Бореля-Лебега про семейство интервалов, пожалуйста.
Аноним 02/01/16 Суб 13:29:27 #486 №322797 
>>322682
Нет, это не частный случай. Множество всех множеств, которые не содержат себя в качестве элемента, и множество всех множеств (т.е. коллективизирующим свойством которого является "P(x) = x есть множество") - это просто два разных множества.

Правильная формулировка парадокса Расселла: не любое свойство является коллективизирующим.
Твой лектор просто лажанулся. Такое бывает.
Аноним 02/01/16 Суб 13:32:15 #487 №322798 
>>322714
Математики изучают разложение объектов на множители. Но такими методами, что у школьника вскипит мозг от одних названий.
Аноним 02/01/16 Суб 13:35:59 #488 №322799 
Есть одна тян область в n-мерном пространстве, задаваемая системой линейных неравенств. Есть ли какой-то простой (ну типа минимизации квадратов) метод (не хочу вдаваться во все эти полиэдры в интернетах). чтобы найти внутреннюю точку области. Неважно ближе к границе области, или дальше. Просто одну точку. Любую. Подозреваю, что что-то похожее на симплекс-метод должно быть (ну или на метод Гаусса, хз). Подскажите, куда копать, пожалуйста.
Аноним 02/01/16 Суб 13:52:00 #489 №322802 
>>322799
Каждое неравенство p(x) < 0 замени линейным уравнением p(x) + y = 0, где y - новая переменная. Получишь систему из m линейных уравнений и n+m переменных. Реши систему и возьми любое решение, где все y положительны. Если такого нет - внутренних точек нет.
Аноним 02/01/16 Суб 15:56:36 #490 №322814 
У кого нибудь есть информация о книжке с курсом матлога от Манина? Нашел ее на английском, а там сказано что это перевод с русского. Обыскал все что только можно, есть только конспекты лекций на печатной машинке (это ясно не они).
Аноним 02/01/16 Суб 16:13:49 #491 №322815 
>>322814
Это конспект?
http://rghost.ru/7NjHZdHrt
http://rghost.ru/6SvdTnsG8
Аноним 02/01/16 Суб 16:41:00 #492 №322821 
>>322752
Любая задача из абстрактной теории групп на самом деле про то как бы упростить выражение в группе с образующими и соотношениями.
В категориях часто берется какое-нибудь пространство морфизмов, а потом n раз применяется какая-нибудь двойственность/сопряженность, что по сути то же упрощение выражения в широком смысле слова.
А вообще вопрос какой-то максимально странный и ебанутый. Примерно в стиле "Набирают ли программисты текст на компьютере?". Да, набирают, например, любая программа это текст. Что дальше? Имеет ли это отношение к программированию? Ну, хуй знает. Это надо уметь делать хоть как-то, впрочем, неясно, каким имбецилом надо, чтобы не уметь это делать. Но при этом документацию к ворду изучать не надо (как и вообще уметь им пользоваться).
Аноним 02/01/16 Суб 16:46:32 #493 №322824 
>>322815

Да. А вот книжка на английском http://avxhome.se/ebooks/science_books/math/2014-05-25-41_201504141540.html
Аноним 02/01/16 Суб 16:50:18 #494 №322826 
>>322824

Хотя это второе издение 2009 года, его скорее всего на русском и нет, но первое датируется 1977, тогда еще он в СССР жил, как так могла получиться что английском учебник издали, а на русском нет?

http://avxhome.se/ebooks/science_books/math/2014-08-26-59_201504180120.html
Аноним 02/01/16 Суб 16:54:59 #495 №322829 
>>322776
Спасибо.
>>322781
Благодарю, хоть я и не школьник.
>>322798
Ну хз, школьники сейчас такие пошли, что мне стыдно становится. Как будто я в школу с товарищами потусить приходил.
>>322821
Хм, интересно, благодарствую. А то я из учебника по общей алгебре вынес, что там люди только описанием поворотов/отображений/подстановок занимаются.
Аноним 02/01/16 Суб 16:59:50 #496 №322831 
>>322796
> Обьясните как для дауна
Это невозможно. Задай конкретный вопрос.
Аноним 02/01/16 Суб 19:42:12 #497 №322985 
>>322802

Спасибо огромное, как я сам не додумался :3

Аноним 02/01/16 Суб 23:11:57 #498 №323079 
14517655170320.png
>>322769
Ну, диметр поделили на пять равных отрезков. Потом один из крайних отрезков взяли и отрезали вместе с частью сферы.
Как то так это должно выглядеть.

>>322772
Это как они треугольник образуют? Там же два прямых угла и дуга.

>>322773
Это же очеидно.
Аноним 02/01/16 Суб 23:13:31 #499 №323081 
>>322714
Второйзамечательный предел.
Аноним 03/01/16 Вск 00:07:46 #500 №323123 
14517688660390.png
>>323079
>Это же очеидно.
Нет, это кривейшая косноязычная формулировка. Если взято из учебника/задачника, то составлявшему надо разбить ебало.
>Это как они треугольник образуют?
Пикрил. ЭТАЖ ИЗИ, БЛЯТЬ, РАДИУС СФЕРЫ ПРОВОДИ ВПОЛТНУЮ К СЕЧЕНИЮ, ЕСЛИ СМОТРЕТЬ НА УСЕЧЕННУЮ СФЕРУ В ПРОФИЛЬ
Не, так-то это мой фейл, надо было сразу приложить чертёж. Дальше разберись сам, а то мне уже спать хочется Всего наилучшего ^_^
Аноним 03/01/16 Вск 00:09:02 #501 №323124 
Аноны, существует ли формальное доказательство коммутативности умножения , или это эмпирический факт.
Аноним 03/01/16 Вск 00:09:20 #502 №323125 
>>323123
>хочется. Всего наилучшего ^_^
Фикс. Это только к тому, что надо отоспаться.
Аноним 03/01/16 Вск 00:12:27 #503 №323127 
>>323123
Сука, ещё расстояние от центра сферы до центра окружности на чертеже неправильное. Не (2/5)R, а (3/5)R. Теперь точно поебу спать.
Аноним 03/01/16 Вск 01:34:38 #504 №323149 
>>323124
Оно не всегда коммутативно. Умножение матриц, например, некоммутативно. Множества натуральных чисел все очевидно (учитывая ассоциативность): а б = а б 1 = а б а^(-1) а = б а, при а а^(-1) = 1 = а^(-1) * а
Аноним 03/01/16 Вск 01:36:54 #505 №323150 
>>323149
Кстати, если кто знает - подскажите, плз, как на местном движке звездочку отображаемую запилить.
Аноним 03/01/16 Вск 05:21:38 #506 №323159 
>>323149
Такое себе доказательство. На самом деле, конечно, нужна индукция.
Аноним 03/01/16 Вск 10:22:58 #507 №323170 
>>323149
Как из аба^(-1)а=> ба, если коммутативность еще не доказана. аба^(-1)а=аб тогда уж.
Аноним 03/01/16 Вск 12:39:24 #508 №323187 
>>323149
Охуенно доказал.
Аноним 03/01/16 Вск 12:52:17 #509 №323190 
Хер с умножением, возьмем сложение. Все равно ведь нужно явно вводить сложение с нулем слева и справа, разве нет?
Аноним 03/01/16 Вск 12:57:17 #510 №323193 
>>323190
Зачем? Минус три прибавить минус четыре будет минус семь. Минус пять прибавить три будет минус два. ЭТОЖЕОЧЕВИДНО, ЧТД.
Аноним 03/01/16 Вск 12:58:07 #511 №323194 
>>323193
>Минус три прибавить минус четыре будет минус семь.
> натуральные числа
Аноним 03/01/16 Вск 13:29:04 #512 №323196 
>>323194
Ты просил доказать, я доказал. Или тебе в кадк с левой и в висок с правой доказать, мразь?
Аноним 03/01/16 Вск 15:23:30 #513 №323211 
>>322304
Ну где же ты
Аноним 03/01/16 Вск 15:36:51 #514 №323214 
>>322126
Ты просто ненавидишь все русское.
Аноним 03/01/16 Вск 16:28:31 #515 №323226 
>>323170
Коммутативность здесь и не используется, это ассоциативность.
Аноним 03/01/16 Вск 16:44:03 #516 №323228 
>>323226
Используется.
Аноним 03/01/16 Вск 17:27:16 #517 №323232 
Матаноны, я немного далековат от математики, потому только сегодня впервые наткнулся на такую штуку:
e^(i pi) = -1

И я такой удивился и задумался. Ведь если возвести обе части выражения в квадрат, то получится следующее:
e^(2i pi) = 1

Что возможно только если степень равна нулю, то бишь:
2i pi = 0
i pi = 0

Почему так?
Полагаю, я где-то ошибся, а если нет, то это как-то абсурдно выглядит. Выходит же тогда что корень -1 равен нулю. Более того, выходит тогда что исходное выражение e^(i pi) = -1 неверно и получается своеобразный парадокс. В общем, объясните, пожалуйста.
Аноним 03/01/16 Вск 17:46:03 #518 №323234 
>>323124
>>323190
Предположим, что у нас уже доказаны ассоциативность сложения и дистрибутивность умножения относительно сложения.

Коммутативность сложения можно доказать по индукции. Пусть для любого m
m + n = n + m.
Тогда
m + (n + 1) = m + 1 + n = 1 + m + n = 1 + n + m = (n + 1) + m.

Умножение целых чисел определяется через повторяющееся сложение. Доказать коммутативность можно по индукции через дистрибутивность:
(n + 1)m = nm + m = mn + m = m(n + 1).
Аноним 03/01/16 Вск 17:54:26 #519 №323236 
>>323232
> Что возможно только если степень равна нулю
Вот здесь ошибка. Это было бы верно, если бы степень была вещественной.
Аноним 03/01/16 Вск 20:20:59 #520 №323260 
>>323232
Интересно, где же тут может быть ошибка, хммм.
Аноним 03/01/16 Вск 20:49:26 #521 №323261 
>>323234
>Коммутативность сложения можно доказать по индукции
А как ты докажешь, что индукция имеет смысл, если ты даже не знаешь определение N?
Аноним 04/01/16 Пнд 00:32:40 #522 №323295 
14518567601260.png
>>323261
Аноним 04/01/16 Пнд 07:53:06 #523 №323348 
>>323261
Никак. Я быдло, которое верит в аксиомы Пеано как в Будду или Аллаха.
Аноним 04/01/16 Пнд 07:53:37 #524 №323349 
>>323348
И в индукцию еще верю просто так.
Аноним 04/01/16 Пнд 14:25:18 #525 №323409 
Вы перекат пилить когда будете? Охуели совсем что ли?
Аноним 04/01/16 Пнд 14:46:02 #526 №323416 
>>323409
пили сам, я бухаю на вписке
водовка картофан порошок наше фсйо
ОП
Аноним 04/01/16 Пнд 15:17:16 #527 №323425 
Если мы будем строить точки на отрезке от 0 до 1 по правилу: для двух соседних точек отмечаем "сумму двоечника" (складываем числители с числителями и знаменатели со знаменателями); то в пределе мы получим все рациональные точки? Это очевидно?
Аноним 04/01/16 Пнд 15:19:40 #528 №323427 
14519099801790.jpg
>>323416
> когда тожы хочица лампава хикковать с будущими филдсовскими лауреатами под веществами на вписоне
Аноним 04/01/16 Пнд 15:34:30 #529 №323429 
>>323425
А как ты, например, дробь 17/1 получишь?
Аноним 04/01/16 Пнд 15:43:42 #530 №323430 
14519114223660.jpg
>>323429
Все рациональные точки на отрезке 0;1
Аноним 04/01/16 Пнд 17:10:32 #531 №323449 
>>323416
Я тоже на вписках сижу в углу и читаю мат тред.
Аноним 04/01/16 Пнд 17:16:59 #532 №323452 
Стоит ли пытаться читать статьи, скажем, на arxiv.org, если я ньюфаг, или пока что не стоит тратить время и нужно читать учебники? Я пока что понимаю только совсем энтрилевельную хуету типа такого

И я пока что не определился, чем хочу заниматься, но математика мне нравится.
Аноним 04/01/16 Пнд 17:17:24 #533 №323453 
>>323452
> типа такого
http://arxiv.org/pdf/1512.03715.pdf
чет ссылка отклеилась
Аноним 04/01/16 Пнд 17:37:58 #534 №323458 
>>323452
>>323453
Да любое знакомство с математикой вредным само по себе не будет, главное не увлекаться. Вообще, конечно хорошо, как я считаю, постепенно всё-таки наметить то что особенно тебе интересно и чем ты хочешь впоследствии заниматься, и выхватывать уже не всю математику - всю ты всё равно никогда не выучишь - а вокруг области, интересующей тебя.
Аноним 04/01/16 Пнд 21:06:17 #535 №323529 
>>323425
Ты заебал со своей бесконечностью между нулем и еденицей.
Аноним 04/01/16 Пнд 21:16:44 #536 №323535 

Квадратру круга теперь вычисляют тут

https://2ch.hk/sci/res/323534.html
comments powered by Disqus

Отзывы и предложения